Probability: Impossible Likely Unlikely Certain Even Chance
Probability: Impossible Likely Unlikely Certain Even Chance
7
Contents
7.1 Random Experiments
and Events
7.2 Classical Definition of
Probability
7.3 Axiomatic Approach
to Probability
7.4 Independent and
Dependent Events
7.5 Random Variables
and Probability
Distributions
7.6 Theoretical Discrete
Distributions
Probability is a measure of uncertainty and deals with the phenomenon of chance or randomness. We quite often
make unknowingly some decisions about probability. For example, when one leaves his house in the morning on a
cloudy day he may decide to take an umbrella with him, even if it is not raining, because he thinks that it may rain
later in the day.
The theory of probability is developed to explain such kind of decisions mathematically. Infact, the theory of
probability has its origins in gambling and games of chance. It is known that a French gambler Chevalier de Mere
approached a French mathematician Blaise Pascal for a solution of a problem concerning gambling. Pascal gave a
solution and subsequently he corresponded with another French mathematician Pierre de Farnat and established
the foundations of the theory of probability. Later Laplace, a French scientist, made first attempt towards giving
some mathematical rigour to the subject of probability. The present day theory of probability is credited to Andrei
Nikolaevich Kolmogorov, a 20th century Russian probabilist, who laid the set theoretic foundations of probability.
Examples
(1) Rolling of an Unbiased Die This experiment can be performed any number of times
Rolling of an unbiased die is a random experiment in under identical conditions.
which all the possible outcomes are 1, 2, 3, 4, 5 and 6, if (2) Tossing of an Unbiased Coin
we denote the six faces of the die with the numbers 1, 2, Tossing of an unbiased coin is a random experiment in
T
3, 4, 5 and 6. The faces of the die may also contain dots which there are only two possible outcomes, namely, Head
in numbers 1, 2, 3, 4, 5 and 6. In any case, we identify (H) and Tail (T). In a particular experiment (tossing of the
the faces of a die with the numbers 1, 2, 3, 4, 5 and 6. coin), the outcome is not known in advance.This experiment
The actual outcome in a particular experiment (rolling can also be performed any number of times under iden-
of the die) is the number that appears on the upper- tical conditions and therefore this is a random experiment.
most face of the die and this is not known in advance.
“Tossing of an unbiased coin till tail appears” is also a random experiment. However, experiments such as
“measuring the acceleration due to gravity using a compound pendulum” is not a random experiment, since the
experiment cannot be repeated under identical conditions and the possible outcomes are not known in advance.
Throughout this chapter, by a coin or die we always mean an unbiased coin (or a fair coin) or unbiased die (or a
fair die) unless otherwise mentioned.
Example
In rolling of a die, consider the events: and E2: occurrence of an odd number.
E1: occurrence of an even number (that is, an T
Then E1 and E2 are exhaustive, mutually exclusive and
even number appears on the upper most equiprobable.
face)
Example
Example 7.1
3 4 5 6 7 8 9
Solution: These sums are not equally likely, as we can 4 5 6 7 8 9 10
observe from the following table. 5 6 7 8 9 10 11
The sum 12 occurs only once, the sum 10 appears 6 7 8 9 10 11 12
thrice and hence “the sum 12 occurs” and “the sum 10
occurs” are not equiprobable (or equally likely).
Example 7.2
In rolling of a die, consider the events Then E1, E2 and E3 are exhaustive, but not mutually
E1: occurrence of an even number exclusive. Why?
E2: occurrence of a prime number
Solution: This is because 2 occurs in both E1 and E2 as
and E3: occurrence of 1 it is both even and prime.
DEFINITION 7.3 Suppose that in a random experiment there are n exhaustive, mutually exclusive and equi-
probable possible outcomes. If m of them are favourable to the happening of an event E,
then the probability of occurance of E (or simply the probability of E) is defined as m/n and is
denoted by P(E). That is,
m
P(E ) =
n
Clearly 0 £ P(E) £ 1.
492 Chapter 7 Probability
Note that since the number of outcomes not favourable to an event E is n - m, the probability of non-occurance of
the event E is (n - m)/n and is denoted by P(Ec). That is,
n-m m
P (Ec ) = = 1 - = P(E )
n n
P (E ) + P (Ec ) = 1
Example 7.3
Suppose that an integer is picked from among 1 to 20 2, 3, 5, 7, 11, 13, 17 and 19 and these are 8 in number.
(both inclusive). What is the probability of picking a Therefore, 8 are favourable to the event of picking a
prime? prime and hence the probability of picking a prime is
8 2
Solution: There are 20 outcomes of the experiment =
of picking an integer. The primes between 1 and 20 are 20 5
Example 7.4
Suppose that 8 dice are thrown. Find the probability that An outcome that none of the dice shows 3 can be
none of the dice shows 3 (on the upper most face). expressed as 8-tuple of integers from the 5-element set
{1, 2, 4, 5, 6} and there are 58 such outcomes. Thus, the
Solution: Any outcome of “throwing 8 dice” can be probability that none of the dice shows 3 is
expressed as an 8-tuple of integers from 1 to 6 and hence 8
the total number of possible outcomes is 68. 58 æ 5 ö
=ç ÷
68 è 6 ø
Example 7.5
Suppose that a bag contains 6 red, 5 black and 4 blue Drawing one blue, one black and one red ball simulta-
balls. Find the probability that three balls drawn simul- neously can be expressed as a triple (a1, a2, a3), where
taneously are one blue, one black and one red. 1 £ a1 £ 4, 1 £ a2 £ 5 and 1 £ a3 £ 6. The number of such
tuples is
Solution: The total number of balls is
4 ´ 5 ´ 6 = 120
6 + 5 + 4 = 15
Thus, the probability that 3 balls drawn simultaneously
Out of these 15 balls, 3 balls can be drawn in 15C3 ways. are one blue, one black and one red is
Therefore, 3 balls can be drawn simultaneously in
120 24
=
15× 14 × 13 455 91
= 455 ways
1× 2 × 3
Example 7.6
A number x is drawn arbitrarily from the set of integers 100. Let A be the event that the number x has the
from 1 to 100, both inclusive. Find the probability that property
100 100
x > 29 - x > 29 -
x x
Now
Solution: When we draw an arbitrary number x from
the set {1, 2, …, 100} the total number of outcomes is 100
x > 29 -
x
7.2 Classical Definition of Probability 493
Û x > 25 or x<4
Û x Î{1, 2, 3} È {26, 27, 28, …, 100}
Example 7.7
DEFINITION 7.4 Let E be an event of a random experiment. Let the experiment be repeated n times out of
which E occurs mn times. Then the ratio
mn
rn =
n
is called the nth relative frequency of the event E. Clearly 0 £ mn £ n and hence 0 £ rn £ 1. If
{rrn} is a convergent sequence, then its limit is defined to be the probability of E and is denoted
by P(E). That is,
mn
P(E) = lim rn = lim
n®¥ n®¥ n
Z+.
It can be easily seen that 0 £ P(E) £ 1, since 0 £ rn £ 1 for all n ÎZ
One can easily observe the following deficiencies in the above statistical definition of probability
1. Repeating a random experiment infinitely many times is impossible.
2. The sequence of relative frequencies must be assumed to be convergent, which may not be true all the time.
494 Chapter 7 Probability
DEFINITION 7.5 1. An outcome of a random experiment is called an elementary eventt or simple event.
2. The set of all outcomes of a random experiment is called the sample space associated with
the experiment. In other words, when x is a random experiment and S is the sample space
associated with x, then every element of S is an outcome of x and any performance of x
results in an outcome that corresponds to exactly one element of S.
Example
Let x be the experiment of rolling a die. Then the sample event, “occurrence of a positive even integer £ 6 on the
space of x is the set S = {1, 2, 3, 4, 5, 6}, where the integers uppermost face of the die” is an event and “occurrence
denote the faces of the die. “Occurrence of a positive of a prime ³ 6 on the uppermost face of the die” is an
integer £ 6 on the uppermost face of the die” is a sure impossible event.
DEFINITION 7.7 1. A set S is set to be countably infinite if it is bijective with the set + of positive integers and,
in this case we can write S = {s1, s2, …, sn, …}.
2. A set S is said to be finite if S is empty or S is bijective with the set {1, 2, …, n} for some
n Î+ and in this case we write S = {s1, s2, …, sn} and say that S is an n-element sett or the
number of elements in S is n.
3. A set S is said to be atmost countable is S is either finite or countably infinite. For example
+, are countably infinite sets.
Note that the sample space S of a random experiment x may be finite or infinite. Throughout our discussions in this
chapter, we take the sample space S of an experiment to be atmost countable; that is, either finite or countably infinite.
For example, the sample space in Examples (1) and (2) below Definition 7.1 are finite and that of the experiment
“Tossing a coin till tail appears” is countably infinite.
DEFINITION 7.8 Let E1, E2, …, En be some events of a random experiment x. That is, E1, E2, …, En are subsets
of the sample space S of x.
1. The events E1, E2, …, En are said to be mutually exclusive if Ei Ç Ej = f for all i ¹ j; that is,
if the happening of an event Ei prevents the happening of any other event Ej, j ¹ i.
2. The events E1, E2, …, En are said to be equiprobable or equally likely if there is no reason
to expect one of them to happen in preference to others.
7.3 Axiomatic Approach to Probability 495
3. The events E1, E2, …, En are said to be exhaustive if E1 È E2 È È En = S; that is, if the
performance of the experiment always results in the occurrence of at least one of Ei ’s.
Note that for any random experiment x whose sample space is S, the set of all possible events of x is Ã(S), the power
set of S (the set of all subjects of S). In the following we introduce Kolmogorov’s axiomatic approach to the theory of
probability.
DEFINITION 7.9 Let x be a random experiment and S its sample space and suppose that S is finite. Then a func-
tion P : Ã(S) ® is called a probability function if it satisfies the following conditions.
1. Positive axiom: P(E) ³ 0 for all E ÎÃ(S)
2. Completeness axiom: Ã(S) = 1
3. Union axiom: If E1 and E2 ÎÃ(S) and E1 Ç E2 = f, then
P(E1 È E2 ) = P(E1 ) + P(E2 )
In this case, for any E ÎÃ(S), P(E) is called the probability of the event E. If E = {s}, then we
write P(s) for P({s}) for simplicity. In the case when S is countably infinite, we have to replace
(3) above by (3¢) given below.
3¢. If {En} is a sequence of mutually exclusive events, then
æ ¥ ö ¥
P ç ∪ En ÷ = å P(En )
è n=1 ø n=1
Example 7.8
Solution: We have
s ÎE1 È E2 if and only if s ÎE1 or s ÎE2 but not
(1) Positive axiom: both.
Thus P is a probability function.
P ( E ) = å P ( s) > 0
s ÎE
T H E O R E M 7.1 The following hold good for any probability function P on Ã(S), where S is the sample space of
a random experiment x.
1. P(f) = 0
2. If Ec is the complementary event of E, then
P (Ec ) = 1 - P (E )
PROOF All these are derived from the axioms (1), (2) and (3) of Definition 7.9.
1. Since S Ç f = f, we have, from Axiom (3) of Definition 7.9 that
0 £ P(E ) £ 1
4. Let E1 Í E2 Í S, Then
E2 = E1 È (E2 - E1 )
and E1 Ç (E2 - E1 ) = f
Therefore, we have
and hence
5. If E1 Í E2 Í S, then
P(E2 ) - P(E1 ) = P(E2 - E1 ) ³ 0
P(E1 ) £ P(E2 )
Note: For any event E, P(E): P(E) (ratio) is called odds in favour of E and the ratio P(E): P(E) is odds against E.
7.3 Axiomatic Approach to Probability 497
T H E O R E M 7.2 Let A, B and C be three events of a random experiment x and P is a probability function on Ã(S),
(A D D I T I O N where S is the sample space of x. Then the following hold.
THEOREM ON 1. P( A È B) + P( A Ç B) = P( A) + P( B)
PROBABILITY)
2. P( A - B) = P( A) - P( A Ç B)
3. P( A È B È C ) = P( A) + P( B) + P(C ) - P( A Ç B) - P( B Ç C ) - P(C Ç A) + P( A Ç B Ç C )
P( A È B) = P[ A - ( A Ç B)] + P( A Ç B) + P[ B - ( A Ç B)]
P( A È B) + P( A Ç B) = P( A) + P( B)
P( A - B) = P[ A - ( A Ç B)] = P( A) - P( A Ç B)
= P( A) + P( B) - P( A Ç B) + P(C ) - P[( A Ç C ) È ( B Ç C )]
= P( A) + P( B) + P(C ) - P( A Ç B) - [ P( A Ç C ) + P( B Ç C ) - P( A Ç C Ç B Ç C )]
= P( A) + P( B) + P(C ) - P( A Ç B) - P( B Ç C ) - P(C Ç A) + P( A Ç B Ç C )
A – (AÇB) A ÇB B – (AÇB)
A B
Note that a probability function P on Ã(S), where S is given to be finite, is completely determined by P(s), s ÎS. If
P(s) is a non-negative real number for each s in a finite set S such that
å P ( s) = 1
sÎS
P( A) = å P( s)
sÎA
In Table 7.1 we give set theoretic descriptions of various events, which will be useful in solving problems on prob-
ability later.
Example 7.9
Example 7.10
Example 7.11
If two numbers are selected randomly from 30 consecu- are 15C2 + 15C2 such ways. Therefore the probability
tive natural numbers, find the probability that the sum of that the sum of the two numbers selected is even is
the two numbers is 15
C2 + 15C2 15 ´ 14 14
(a) even. 30
= =
C2 435 29
(b) odd.
The event “a + b is odd” is complementary to the
Solution: Two numbers can be selected from 30 in event “a + b is even” and therefore, the probability
30
C2 = 435 ways. Since the given 30 numbers are consecu- that the sum of the two numbers selected odd is
tive, there are 15 even and 15 odd numbers among them.
14 15
(a) For a + b to be even, it is necessary and sufficient 1- =
that either both a and b are even or both odd. There 29 29
DEFINITION 7.10 Let x be a random experiment and A and B be two events of x. Then the event “occurrence
of B after the occurrence of A” is called a conditional eventt and is denoted by B/A
/ . Similarly,
the event “occurrence of A after the occurrence of B” is denoted by A/B.
DEFINITION 7.11 Let A and B be two events of a random experiment x. The conditional probability of occur-
rence of A after the occurrence of B is defined by
P( A Ç B)
P( B)
where P(B) > 0 and is denoted by P(A/B). That is,
P( A Ç B)
P( A / B) = , when P( B) > 0
P( B)
P( B Ç A)
P( B / A) = , when P( A) > 0
P( A)
QUICK LOOK 1
Example 7.12
A pair of fair dice is thrown. Find the probability that from 1 to 6. Let A be the event “2 appears on either of
either of the dice shows 2 if the sum is 6. the dice” and B be the event “sum is 6”. We want to find
P(A/B). Note that
Solution: The sample space of the experiment
“throwing a pair of fair dice” consists of 36(= 6 ´ 6) A = {(2, b)| 1 £ b £ 6} È {(a, 2)| 1 £ a £ 6}
ordered pairs (a, b), where a and b can be any integers and B = {(1, 5), (2, 4), (3, 3), (4, 2), (5, 1)}
500 Chapter 7 Probability
Also, So
A Ç B = {(2, 4), (4, 2)} P( A Ç B) 2 / 36 2
P( A / B) = = =
Therefore P( B) 5 / 36 5
5 2
P( B) = and P( A Ç B) =
36 36
Example 7.13
T H E O R E M 7.3 Let A and B be two events of a random experiment such that P(A) > 0 and P(B) > 0. Then
(M U L T I P L I C A T I O N
P( A Ç B) = P( A)P( B / A) = P( B)P( A / B)
THEOREM ON
CONDITIONAL
PROBABILITY)
PROOF This is an immediate consequence of the definition of the conditional probabilities P(A/B) and
/ ). This can be extended to any finite number of events E1, E2, …, En, by using induction
P(B/A
on n. ■
Then
æ n-1
ö
P ( E1 Ç E2 Ç Ç En ) = P(E1 )P ( E2 / E1 ) P ( E3 /(E1 Ç E2 )) P ç En / æ Ç Ei ö ÷
è è i =1 ø ø
Example 7.14
A bag contains 20 identical balls of which 8 are black and then the probability that the third ball drawn is blue is 10/18.
12 are blue. Three balls are taken out at random from Therefore, by Corollary 7.1, the required probability is
the bag one after the other without replacement. Find
12 11 10 11
the probability that all the three balls drawn are blue. × × =
20 19 18 57
Solution: The probability that the first ball drawn is blue Note: If the drawn ball is replaced every time, then the
is 12/20, since there are 12 blue balls among 20 balls in the probability is
bag. If the first ball is blue, then the probability that the
3 3
second ball drawn is blue is 11/19, since 11 of the remaining æ 12 ö æ 3 ö
19 are blue. Similarly, if the first two balls drawn are blue, çè ÷ø = çè ÷ø
20 5
7.4 Independent and Dependent Events 501
Example 7.15
A jar contains 10 white balls and 6 blue balls, all are of since 10 out of 10 + 6 balls are white. But, after one ball
equal size. Two balls are drawn without replacement. is chosen, there remain 9 white balls and 6 blue balls.
Find the probability that the second ball is white if it is Therefore the required probability is
known that the first is white.
P(E1 Ç E2 )
P(E2 / E1 ) =
Solution: Let E1 be the event “the first ball drawn is P(E1 )
white” and E2 be the event “the second ball drawn is 10 9
white again. Then, .
9 3
= 16 15 = =
10 10 / 16 15 5
P(E1 ) =
16
T H E O R E M 7.4 Let S be the sample space of a random experiment x and P be a probability function on Ã(S), the
set of all events of x. Let A ÎÃ(S) such that P(A) > 0. Define PA : Ã(S) ® by
PA (E) = P(E / A)
for any E ÎÃ(S). Then PA is also a probability function.
PROOF Recall that
P(E Ç A)
PA (E) = P(E / A) =
P( A)
Since P(A) > 0 and P(E Ç A) ³ 0, PA(E) ³ 0 for all E ÎÃ(S). Also,
P(S Ç A) P( A)
PA (S) = = =1
P( A) P( A)
since A Í S. Further, let E1 and E2 be two mutually exclusive events (i.e., E1 and E2 Í S and
E1 Ç E2 = f). Then,
P[(E1 È E2 ) Ç A]
PA (E1 È E2 ) =
P( A)
P[(E1 Ç A) È (E2 Ç A)]
=
P( A)
P(E1 Ç A) + P(E2 Ç A)
= (since E1 Ç A Ç E2 Ç A = f )
P( A)
= PA (E1 ) + PA (E2 )
DEFINITION 7.12 Let x be a random experiment and S its sample space. Let P be a probability function on
Ã(S). Two events A and B are said to be independent of each otherr if the occurrence of one
of them does not influence the occurrence of the other. That is, B is independent of A if
P( B) = P( B / A)
If A and B are not independent, then we say that they are dependent.
T H E O R E M 7.5 The following are equivalent to each other for any two events A and B of a random experiment.
1. A and B are independent of each other.
2. P(B) = P(B/A
/ )
502 Chapter 7 Probability
3. P(A) = P(A/B)
4. P(A Ç B) = P(A) P(B)
PROOF (1) Û (2) follows from Definition 7.12.
(2) Û (3) Û (4) follow from the multiplication theorem (Theorem 7.3), where we have
P( A Ç B) = P( A)P( B / A) = P( B)P( A / B) ■
Example 7.16
Let A and B be independent events with P(A) = 0.6 and (2) P(A/B) is given by
P(B) = 0.2. Find the following.
P( A Ç B) 0.12 3
(1) P(A Ç B) P( A / B) = = =
P( B) 0.2 5
(2) P(A/B)
P(B/A
/ ) is given by
(3) P(B/A
/ )
(4) P(A È B) P( A Ç B) 0.12 1
P( B / A) = = =
P( A) 0.6 5
Solution: From Theorem 7.5, we have
P(A È B) is given by
(1) P(A Ç B) is given by
P( A È B) = P( A) + P( B) - P( A Ç B)
3
P( A Ç B) = P( A)P( B) = (0.6)(0.2) = 0.12 = 17
25 = 0.6 + 0.2 - 0.12 = 0.68 =
25
Example 7.17
Example 7.18
A bag B1 contains 3 white balls and 4 black balls and Let A be the event that the ball chosen is white. Then,
another bag B2 contains 2 black balls and 4 white balls.
3 4 2
A bag is drawn at random and a ball is drawn at random P( A / E1 ) = and P( A / E2 ) = =
from it. Find the probability that the ball drawn is white. 7 6 3
Since E1 and E2 are exhaustive and mutually exclusive,
Solution: Let E1 and E2 be the events of choosing B1 it follows that
and B2, respectively. Then,
A = ( A Ç E1 ) È ( A Ç E2 )
1
P(E1 ) = = P(E2 ) and ( A Ç E1 ) Ç ( A Ç E2 ) = f
2
7.4 Independent and Dependent Events 503
Therefore 1 3 1 2
= × + ×
2 7 2 3
P( A) = P( A Ç E1 ) + P( A Ç E2 )
3 1 23
= P(E1 )P( A / E1 ) + P(E2 )P( A / E2 ) = + =
14 3 42
T H E O R E M 7.6 Let E1, E2, E3, …, En be mutually exclusive and exhaustive events with non-zero probabilities of
(T O T A L a random experiment x. Then for any event A connected with x,
PROBABILITY) n
P( A) = å P(Ei )P( A / Ei )
i =1
PROOF We have
A= AÇS
= A Ç (E1 È E2 È È En )
= ( A Ç E1 ) È ( A Ç E2 ) È È ( A Ç En )
n n
P( A) = å P( A Ç Ei ) = å P(Ei )P( A / Ei )
i =1 i =1 ■
T H E O R E M 7.7 Let E1, E2, …, En be exhaustive and mutually exclusive events of a random experiment x and S be
(B A Y E S ’ the sample space of x with a probability function P on Ã(S) such that P(Ei) > 0 for all 1 £ i £ n.
THEOREM) Then for any event A of the random experiment x,
P ( Ej ) P ( A / Ej )
P(Ej / A) = n
å P (E )P ( A / E )
i =1
i i
£ j £ n.
PROOF By hypothesis E1, E2, …, En are exhaustive and mutually exclusive and hence
E1 È E2 È En = S
and Ei Ç Ej = f for i¹j
Also, it is given that P(Ei) > 0 for all 1 £ i £ n. For any event A (i.e., A Í S), we have
A = A Ç S = A Ç æ È Ei ö = È( A Ç Ei )
n n
è i =1 ø i =1
A Ç Ei Ç A Ç Ej = A Ç Ei Ç Ej = f for i ¹ j we have
n
P( A) = å P( A Ç Ei )
i =1
n
= å P(Ei )P( A / Ei ) (by Theorem 7.3)
i =1
å P(E )P( A / E )
i =1
i i
■
504 Chapter 7 Probability
Example 7.19
Three bags, B1, B2 and B3 contain balls as given in Table 7.2. Having choosen the bag Bi, the probability of choosing a
red ball is P(R/Ei) and is given by
Table 7.2 Example 7.19
2 4 2
Red White Black P(R / E1 ) = , P(R / E2 ) = and P(R / E3 ) =
5 9 9
B1 2 2 1 We want to find the probability P(E2/R). By Bayes’
B2 4 3 2 theorem, we get that
B3 2 4 3
P(E2 )P(R /E2 )
P(E2 /R) =
P(E1)P(R / E1) + P(E2 )P(R /E2 ) + P(E3 )P(R /E3)
A die is thrown. B1 is chosen if either 1 or 2 turns up,
B2 is chosen if either 3 or 4 turns up and B3 is chosen if 1 4
´
either 4 or 5 turns up. Having chosen a bag in this way, a = 3 9
ball is chosen at random from this bag. If the ball chosen æ 1 2ö æ 1 4ö æ 1 2ö
çè ´ ÷ø +ç ´ ÷ +ç ´
è 3 9ø è 3 ÷
is of red colour, find the probability that it comes from 3 5 9ø
bag B2?
4
Solution: Let Ei be the event of choosing the bag Bi, for = 27
i = 1, 2, 3. Let R be the event of choosing a red ball. Then æ 18 + 20 + 10 ö
çè 3 ´ 5 ´ 9 ÷ø
1
P(Ei ) = for i = 1, 2, 3
3 4 3´5´9 5
= ´ =
27 48 12
DEFINITION 7.13 Let S be the sample space of a random experiment. Then any real-valued function defined on
S is called a random variable on S. That is, a random variable of S is simply a function J :S ® R.
Examples
(1) Let S be the sample space of the experiment (3) Let S be the sample space of the experiment “tossing
That is, S = {1, 2, 3, 4, 5, 6}. Define
“rolling a fair die”. T three fair coins simultamously”.
J :S ® by Then
ïì1 if s is even S = {HHH, HHT, HTH, HTT, THH, THT, TTH, TTT}
J( s) = í
îï0 if s is odd Define J : S ® R by
That is, J (1) = J (3) = J (5) = 0 and J (2) = J (4) =
T J (s) = The number of heads in s
J (6) = 1. Then J is a random variable on S.
For example, J (HHH) =3, J (HHT) = 2, J (THT) = 1,
(2) Let S be the sample space of the experiment “tossing J (TTT) = 0. Then J is a random variable on S.
a fair coin”. That is, S = {H, T
T}, where H = Head and
T = Tail. Define J : S ® R by
J ( H ) = 1 and J (T ) = 0
Then J is a random variable on S.
For any random experiment x and its sample space S, we prove in the following that any probability function on
Ã(S) and a random variable on S give rise to a probability function on P().
7.5 Random Variables and Probability Distributions 505
T H E O R E M 7.8 Let S be the sample space of a random experiment. Let P : Ã(S) ® be a probability function
and J : S ® be a random variable. Define P¢ : Ã() ® by
P¢(T) J-1(T))
T = P(J T for any T Í ,
where J-1(T)
T = {s ÎS | J (s) ÎT}.
T Then P¢ is a probability function on Ã() and is called the prob-
ability function induced by P and J.
PROOF Since P(E) ³ 0 for all E Í S, we have
J -1 ( A) Ç J -1 ( B) = J -1 ( A Ç B) = J -1 (f ) = f
and J -1 ( A È B) = J -1 ( A) È J -1 ( B)
and therefore
P ¢( A È B) = P[J -1 ( A È B)]
= P[J -1 ( A) È J -1 ( B)]
= P[J -1 ( A)] + P[J -1 ( B)]
= P ¢( A) + P ¢( B)
Thus P¢ is a probability function on Ã(). ■
Example
It is a convention to use the letter X to denote a random variable and as such a random variable is simply a func-
tion X from the sample space S into the real number system. Here afterwards, we use X to denote a random variable.
506 Chapter 7 Probability
DEFINITION 7.14 Let S be the sample space of a random experiment and X : S ® be a random variable.
Then, to each real number r, the event
X -1 ((-¥, r ]) = {s Î S | X ( s) £ r}
F (r + 0) = lim F ( s) = F (r )
s® r
s >r
DEFINITION 7.16 A random variable whose range is atmost countable (i.e., finite or countably infinite) is
called a discrete random variable. A random variable which is not discrete is called a contin-
uous random variable.
The random variables given in all the above examples are discrete. Though there are certain continuous random
variables, we confine our discussion to the discrete random variables only. In the following we introduce the notions
of mean, variance, standard deviation and probability distribution of discrete random variables.
7.5 Random Variables and Probability Distributions 507
DEFINITION 7.17 Let X be a discrete random variable on the sample space S of a random experiment and let
Z+,
the range of X be {x1, x2, …, xn, …}. Let P be a probability function on Ã(S). For each n ÎZ
let P(X = xn) be defined by
P( X = xn ) = P( X -1 {xn })
m = å xn P( X = xn )
n
QUICK LOOK 2
From the points given in Definition 7.17, we have = å xn2 pn - 2 m 2 + m 2 (since å xn pn = m and å pn = 1)
n n n
s 2 = å ( xn - m )2 pn
n = å x pn - m
2
n
2
n
= å xn2 pn - 2 m å xn pn + m 2 å pn
n n n Therefore
s 2 + m 2 = å xn2 pn
n
Example 7.20
The probability distribution of a random variable X is = 0.25 + 1.00 + 2.25 + 4 + 6.25 + 9 + 12.25
given in Table 7.3.
= 35
Table 7.3 Example 7.20 The variance is given by
X=x 1 2 3 4 5 6 7 7
Example 7.21
DEFINITION 7.18 A random variable X is said to follow binomial distribution, or simply, it is a binomial variate
with parameters n and p if
P(X = k) = nCk pkqn-k for k = 0, 1, 2, …, n
and this is described by writing X ~ B(n, p).
Note that P(X = k) are nothing but the terms of the binomial expansion of (p( + q)n and this justifies the name ‘bino-
mial distribution”. Also, recall that q = 1 - p.
The following can be easily proved by using the results in Chapter 7, Binomial Theorem, Vol. 1.
T H E O R E M 7.10 If X ~ B (n, p), then the mean m and the variance s 2 of X are equal to np and npq, respectively.
Notice that a binomial distribution takes place under the following experimental conditions.
1. Each trial results in two mutually exclusive outcomes, termed as success and failure.
2. The number n of trials is finite.
3. The trials are independent of each other.
4. The probability p of success is constant for each trial.
l k -l
P( X = k ) = e for k = 0, 1, 2, …
k!
l > 0 is a constant. This is called the Poisson law. Table 7.5 shows the Poisson distribution.
T H E O R E M 7.11 Let X be a Poisson random variate with parameter l. Then the mean and variance of X are both
equal to l.
Note that the Poisson distribution can be used under the following experimental conditions.
1. Each trial results in two mutually exclusive outcomes, termed as success and failure.
2. The number n of trials is sufficiently large.
3. The probability p of success is very small.
4. The trials are independent of each other.
510 Chapter 7 Probability
QUICK LOOK 3
The Poisson distribution can be used, for instance, in 3. The number of vehicles passing a given spot per
problems like the following: minute during the peak hours of a day in a given
1. The number of defective items in a packing city.
2. The number of deaths from non-epidemic diseases
such as heart attack or cancer or snake bites
Example 7.22
Eight coins are tossed simultaneously. Find the probability r = 0, 1, 2, …, 8. Therefore, the probability of getting
of getting atleast six Heads, using the binomial distribution. atleast 6 heads is
Solution: Let P( X ³ 6) = P( X = 6) + P( X = 7) + P( X = 8)
8 8 8
1 æ 1ö æ 1ö æ 1ö
p = the probability of getting a head = = 8C6 ç ÷ + 8C7 ç ÷ + 8C8 ç ÷
2 è 2ø è 2ø è 2ø
1 7 1 1 37
and q = the probability of getting a tail = = + + =
2 64 32 256 256
The probability of getting r heads in a random toss is
r 8-r 8
æ 1ö æ 1ö æ 1ö
P ( X = r ) = 8 Cr ç ÷ ç ÷ = 8 Cr ç ÷
è 2ø è 2ø è 2ø
Example 7.23
In a book of 750 pages, there are 500 typographical The probability of r errors per page is
errors. Assuming Poisson law for the number of errors r
per page, find the probability that a random sample of l r - l æ 2 ö 1 -2 / 3
P( X = r ) = e =ç ÷ e
5 pages will contain no error. r! è 3ø r!
Therefore
Solution: The average member of errors per page in
the book is P( X = 0) = e-2 / 3
500 2 The required probability that a random sample of
l= =
750 3 5 pages will contain no error is
( P( X = 0))5 = (e-2 / 3 )5 = e-10 / 3
Example 7.24
l -l l2 -l
e = e
1! 2!
Worked-Out Problems 511
WORKED-OUT PROBLEMS
Single Correct Choice Type Questions
1. A, B and C are three athletes running in a race. If Solution:
the probability of A winning is twice as likely to win real number and sum of all the probabilities is equal to 1,
as probability of B and that of B is as likely to win as the correct answer is (A).
of C, then the probability of A’s win is Answer: (A)
1 2 3 4
(A) (B) (C) (D)
7 7 7 7 4. A die is so weighed such that the probability of a
number appearing when tossed is proportional to the
( ) = 2P(B) and P(B) = 2P(C).
Solution: By hypothesis P(A number on the face. Then, the probability of a prime
Since numbered face to appear is
P( A) + P( B) + P(C ) = 1 4 5 10 11
(A) (B) (C) (D)
7 21 21 21
we have
Solution: Let p be the constant of proportionality so
4 P(C ) + 2 P(C ) + P(C ) = 1
that
Therefore P(K ) = Kp for K = 1, 2, 3, 4, 5, 6
1
7 P(C ) = 1 or P(C ) = Sum of the probabilities = 1. This implies
7
1
So 21 p = 1 Þ p =
21
4
P( A) = 4 P(C ) = Let E be the event of a prime number face. That is
7 E = {2, 3, 5}. Then
Answer: (D)
10
P(E ) = 2 p + 3 p + 5 p =
2. In Problem (1), the probability that one of B or C will 21
win is Answer: (C)
2 3 4 5
(A) (B) (C) (D) 5. A and B are two events of a random experiment. If
7 7 7 7
P( A È B) = 7 / 8, P( A Ç B) = 1/ 4 and P( A) = 5 / 8, then
Solution: The events that either of B or C will win are P( A Ç B) is equal to
mutually exclusive events so that P(B Ç C) = 0. Hence 1 1 1 3
(A) (B) (C) (D)
P( B È C ) = P( B) + P(C ) 8 4 3 8
2 1 3 Solution: We have
= + =
7 7 7
P( A Ç B) = P( A - B) = P( A) - P( A Ç B)
Answer: (B)
æ 5ö 1
= ç1 - ÷ -
3. Let S = {x1, x2, x3, x4} be a four-element sample space. è 8ø 4
Which of the following functions defines a probability
3 1 1
function on S? = - =
8 4 8
1 1 1 1
(A) P( x1 ) = , P( x2 ) = , P( x3 ) = , P( x4 ) =
4 8 2 8 Answer: (A)
1 1 1 1
(B) P( x1 ) = , P( x2 ) = , P( x3 ) = , P( x4 ) = 6. Two dice of different colours are thrown at a time.
3 5 4 2
The probability that the sum of the faces appeared is
1 1 1 1 either 7 or 11 is
P( x1 ) = , P( x2 ) = - , P( x3 ) = , P( x4 ) =
2 3 4 2 7 4 2 5
1 1 2 (A) (B) (C) (D)
P( x1 ) = 0, P( x2 ) = , P( x3 ) = , P( x4 ) = 36 9 3 9
2 3 3
512 Chapter 7 Probability
(C)
1296
(D)
1396 E = (b1 Ç b2 Ç w3 ) È (b1 Ç w2 Ç b3 ) È (w1 Ç b2 Ç b3 )
270725 270725
Each of the events in the union is mutually exclusive and
Solution: Total number of outcomes = 52C4 = 270725 independent. Hence
E = Event of exactly two spade cards and exists two aces.
A = Event of 1 spade ace, 1 non-spade ace and 1 spade P(E) = P(b1 Ç b2 Ç w3 ) + P(b1 Ç w2 Ç b3 )
card and 1 non-spade card + P(w1 Ç b2 Ç b3 )
B = Event of 2 non-spade aces and 2 spade cards.
= P(b1 )P(b2 )P(w3 ) + P(b1 )P(w2 )P(b3 )
1(3C1 )(12C1 )(36C1 ) 3 ´ 12 ´ 36 1296
P( A) = 52
= = + P(w1 )P(b2 )P(b3 )
C4 270725 270725
2 1 2 2 3 3 1 1 3
3
C2 ´ 12C2 3 ´ 66 198 = × × + × × + × ×
P( B) = 52
= = 3 4 5 3 4 5 3 4 5
C4 270725 270725
25 5
= =
Now E = A È B and A Ç B = f. This implies 60 12
P(E) = P( A È B) Answer: (D)
1494 10. A class contains 20 boys and 20 girls of which half the
= P( A) + P( B) =
270725 boys and half the girls have cat eyes. If one student
Answer: (A) is selected from the class, the probability that either
the student is a boy or has cat eyes is
8. 1 3 3 2
(A) (B) (C) (D)
husband) are standing in a room. Four people are 2 4 8 3
chosen at random. The probability that the selection
contains exactly one couple is Solution: Let A be the event of a boy and B the event
of having cat eyes. So
1 1 16 17
(A) (B) (C) (D)
11 33 33 33 20 1
P( A) = =
40 2
Solution: From 12 people, 4 can be selected in
C4 = 495 ways.
12
and P( B) =
20 1
=
E = Event of the selection contains exactly one couple. 40 2
Therefore the selection must contain one couple and the
rest 2 must be non-couple. This can be done in
10 1
6
C1 ´ 2(5C1 ´ 4 C1 ) = 6 ´ 40 = 240 ways P( A Ç B) = =
40 4
Worked-Out Problems 513
3 5 9 1
(A) (B) (C) (D) Women Men No. of selections
4 8 40 4
3 2 6
C3 ´ 5C2
Solution: We have by Theorem 7.3
4 1 6
C4 ´ 5C1
P( A Ç B) = P( A)P ( B / A) = (0.3)(0.5) = 0.15
5 0 6
C5 ´ 5C0
Now
Therefore
P( A È B) = P( A) + P( B) - P( A Ç B)
= 0.3 + 0.6 - 0.15
6
C5 ´ 5C2 + 6C4 ´ 5C1 + 6C5 ´ 5C0
P(E ) = 11
C5
= 0.75
6 ´ 10 + 15 ´ 5 + 6 ´ 1
Also =
462
P( A Ç B) 141
P( A / B) = =
P( B) 462
P( A È B) Answer: (A)
=
P( B)
15. Each of two bags A and B contain n cards numbered
1 - P( A È B) 1 to n. One card from each of A and B is drawn.
=
1 - P( B) The probability that the card drawn from A bears
number smaller than the number on the card drawn
1 - 0.75
= from B is
1 - 0.6
n+1 n-1 n+1 n-1
(A) (B) (C) (D)
0.25 250 5 2n 2n n n
= = =
0.4 400 8
Answer: (B)
514 Chapter 7 Probability
1 P(E Ç F Ç G) = P(G - (E Ç F ))
2 P(E1 ) = 1 - P(E3 ) = 1 -
n = P(G - (E È F ))
n-1 = P(G - G Ç (E È F ))
Þ P(E1 ) =
2n
= P(G) - P((G Ç E) È (G Ç F ))
Answer: (B) (See Theorem 7.2)
= P(G) - P(G Ç E) - P(G Ç F )
16. A, B, C are three events such that P(A) = 0.3,
P(B) = 0.4, P(C) = 0.8, P(A Ç B) = 0.08, P(C Ç A) = + P(G Ç E Ç F ) (By Theorem
m 7.2)
0.28, P(A Ç B Ç C) = 0.09, and P(A È B È C) ³ 0.75. = P(G) - P(G)P(E) - P(G)P(F ) + 0
Then P(B Ç C) belongs to the interval:
(∵ P(E Ç F Ç G) = 0 and E, F
(A) [0.11, 0.23] (B) [0.23, 0.48]
and G are pairwise independent)
(C) [0.13, 0.25] (D) [0.15, 0.20]
= P(G)(1 - P(E) - P(F ))
Solution: We have
= P(G)( P(E) - P(F ))
P( A È B È C ) = (å P( A)) - P( A Ç B) - P( B Ç C )
Hence
- P(C Ç A) + P( A Ç B Ç C )
P(E Ç F Ç G)
= (0.3 + 0.4 + 0.8) - 0.08 - P( B Ç C ) P((E Ç F )/G)) =
P(G)
- 0.28 + 0.09 (7.1)
P(G)[ P(E) - P(F )]
= = P (E ) - P (F )
Therefore P(G)
P( B Ç C ) = 1.23 - P( A È B È C ) ³ 1.23 - 1 Answer: (C)
0.23 £ P( B Ç C ) £ 0.48 3× 4 2
= =
9 × 10 15
Answer: (B)
Answer: (A)
Worked-Out Problems 515
P(E / F ) + P(E / F ) C
P(E Ç F ) P(E Ç F )
= + FIGURE 7.2 Single correct choice type question 20.
P(F ) P(F )
Answer: (A)
P(E Ç F )P(F ) + P(E Ç F )P(F )
=
P(F )P(F ) 21. If three distinct natural numbers are chosen randomly
from the first 100 natural numbers, then the probability
P(E Ç F )(1 - P(F )) + P(E Ç F )P(F ) that all three of them are divisible by both 2 and 3 is
=
P(F )P(F ) 4 4 4 4
(A) (B) (C) (D)
P(F )[ P(E Ç F ) - P(E Ç F )] + P(E Ç F ) 25 35 33 1155
= ¹1
P(F )P(F ) Solution: A number is divisible by 6, only when it is
So (C) is not correct. divisible by both 2 and 3. Therefore the number of multi-
ples of 6 (<100) is 16 (they are 6, 12, 18, …, 96).
516 Chapter 7 Probability
From 100 numbers three are selected in 100C3 ways. 23. A natural number x is selected at random from the
From 16 numbers 3 are selected in 16C3 ways. first 100 natural numbers. The probability that
Therefore probability is x + (100 / x) > 50 is
16
C3 16 3 197 11 9 13 3
= ´ (A) (B) (C) (D)
100
C3 3 13 100 20 20 20 4
Solution: We have
16 ´ 15 ´ 14 4
= = 100
100 ´ 99 ´ 98 1155 x+ > 50
x
Alternate Solution
Since the three numbers are distinct, we can select the Û ( x - 25)2 > 525
three one after other without replacement. Therefore the Û | x - 25| > 22
probability is
Hence x < 3 or x > 47. So the number of favourable
16 ´ 15 ´ 14 4
= cases to x is 2 + 53 = 55. The required probability is
100 ´ 99 ´ 98 1155
55 11
Answer: (D) =
100 20
22. Let A, B and C be mutually independent events. Answer: (A)
Consider the following two statements.
Statement I: A and B È C are independent. 24. The first 12 letters of English alphabet are written
Statement II: A and B Ç C are independent. in a row at random. The probability that there are
Then, exactly four letters in between A and B is
(A) both I and II are true 5 1 7 1
(A) (B) (C) (D)
(B) only I is true 66 22 66 11
(C) only II is true Solution: A and B can be arranged in 12P2 = 11 ´ 12
(D) both I and II are false ways. Since we want 4 letters in between A and B, the
order of the four letters appearing has no importance.
Solution: We know that A and B can take the following places.
A Ç ( B È C ) = ( A Ç B) È ( A Ç C )
Place for A Place for B
Now
1 6
P( A Ç ( B È C )) 2 7
= P( A Ç B) + P( A Ç C ) - P( A Ç B Ç C ) 3 8
4 9
= P( A)P( B) + P( A)P(C ) - P( A)P( B)P(C )
5 10
= P( A)[ P( B) + P(C ) - P( B)P(C )] 6 11
= P( A)[ P( B) + P(C ) - P( B Ç C )] 7 12
= P( A)P( B È C )
A and B can be interchanged. Therefore required prob-
Therefore Statement I is true. Again ability is
P( A Ç ( B Ç C )) = P( A)P( B)P(C ) 14 7
=
= P( A)P( B Ç C ) 11 ´ 12 66
Alternate Solution
Hence A and B Ç C are independent. So Statement II is
12 letters can be arranged in 12 ways. In between
also true.
A and B, four letters can be arranged in 10P4 ways. Now
Answer: (A) 6 letters for whom A and B are at extreme positions
together with 6 other letters can be arranged in 7 ways.
Worked-Out Problems 517
Further A and B can be interchanged. Therefore the 27. Each of the letters A, B, C, D, E and F are, respec-
required probability is tively, written on six cards (one letter on one card only)
and they are well-shuffled and then the top four cards
2 ´ 10 P4 ´ 7 2 ´ 10 ´ 7 are turned face up without changing their order. Then
=
12 6 ´ 12 the probability that they form the word “DEAF” is
2´7 7 1 1
= = (A) (B) (C) 0.03 (D) 0.13
11 ´ 12 66 120 360
Answer: (C)
Solution: Total number of arrangements by cards = 6P4 =
360. In only one way, the letters form the word DEAF.F
25. If the letters of the word UNIVERSITY are arranged
Therefore
in a random order, the probability that all the vowels
come together is 1
Probability =
1 2 1 1 360
(A) (B) (C) (D)
15 15 5 30 Answer: (B)
Solution: The word UNIVERSITY consists of 10
28. A boy remembers all but the last digit of his girlfriend’s
letters out of which the vowels are e, i, i, u. The 10 letters
mobile number. He randomly chooses a digit from
can be arranged in 10 / 2 ways (since there two identical
0 to 9 (including 0 and 9). If he attempts two times, the
letters i). Since all the vowels are to be together, treat all
probability that he reaches her at least once is
the four written as a single object. The 6 + 1 = 7 objects
can be permuted in (A) 0.2 (B) 0.3 (C) 0.02 (D) 0.03
Solution: Let
æ 4ö
7 ç ÷ ways E = Event that the boy makes correct dial at least once.
è 2ø
E = Event of failing in both attempts.
(since 4 vowels can be arranged among themselves in In the first attempt, the probability of failing is 9/10. In the
4 / 2 ways). Therefore required probability is second attempt, the probability of failing is 8/9. Therefore
7( 4 / 2) 24 1 1 9 8 4
= = = P(E ) = ´ =
10 / 2 8 ´ 9 ´ 10 3 ´ 10 30 10 9 5
Answer: (D) 4 1
P(E ) = 1 - P(E ) = 1 - = = 0.2
5 5
26. A cubical die has four blank faces, one face marked Answer: (A)
2 and another marked 3. If the die is thrown 5 times,
the probability that the sum is 12 is 29. The probability of the birth dates of all 6 persons to
5 5 5 5 fall in only two different months is
(A) (B) (C) (D)
1296 1294 2596 2592 341 341 341 541
(A) (B) (C) (D)
Solution: Total number of outcomes is 126 125 124 126
Solution: Since the birth date of any person can fall in
6 ´ 6 ´ 6 ´ 6 ´ 6 = 65
anyone of the 12 months, the number of total outcomes
Let is 126. Let
E = sum of the faces is 12. E: Event that the birth dates of all 6 fall in two different
Favourable cases to E : 2 three times, 3 two times or months.
3 four times. This is possible in Number of possibilities of 6 persons’ birth dates to fall
in two different months (say February and March) is
5 5 26 -2 (this 2 corresponds to either all their birth dates fall
+ ways
23 4 in February or all fall in March). The number of ways
that we can select 2 months out of 12 is 12C2. Therefore
Therefore
12
C2 (26 - 2) 66 ´ 62 11 ´ 31 341
10 + 5 15 5 5 P(E ) = = = = 5
P(E ) = = 4 = = 126 126 125 12
65
6 ´6 2´6 4
2592
Answer: (B)
Answer: (D)
518 Chapter 7 Probability
æ 1ö 1 Answer: (C)
and P ( A /C ) = 3 ç ÷ =
è 6ø 2
32. Let A, B and C be three events such that p = P(exactly
Now one of A or B) = P(exactly one of B or C) = P(exactly
A = A Ç ( B È C ) = ( A Ç B) È ( A Ç C ) one of C or A) and P(A, B, C simultaneously) = p2
where 0 < p < 1/ 2. Then P(at least one of A, B or C)
Since B and C are mutually exclusive we have is equal to
P( A) = P( A Ç B) + P( A Ç C ) 3 p + 2 p2 2 p + 3 p2
(A) (B)
2 2
= P( B)P( A / B) + P(C )P( A / c)
2 p + 3 p2 3 p + 2 p2
1 1 1 1 1+ 3 1 (C) (D)
= ´ + ´ = = 4 4
2 6 2 2 12 3
( - B) È (B - A).
Solution: Exactly one of Aor Bmeans (A
Therefore By hypothesis
1 2 P(( A - B) È ( B - A)) = p
P( A) = 1 - =
3 3
P(( B - C ) È (C - B)) = p
Hence odds in favour (see Note under Theorem 7.1) is
P(E): P(E) = 1 : 2. P((C - A) È ( A - C )) = p
Answer: (C)
Therefore
31. An urn contains m white balls and n black balls. p = P(( A - B) È ( B - A)) = P( A - B) + P( B - A)
A ball is drawn at random and put back into the urn
along with k additional balls of the same colour as = [ P( A) - P( A Ç B)] + [ P( B) - P( B Ç A)]
that of the drawn ball. A ball is again taken out at
So
random. The probability that it is white is
m n P( A) + P( B) - 2 P( A Ç B) = p (7.4)
(A) (B)
m+n m+n
Similarly
m+n+k mn
(C) (D)
mn + k mn + k P( B) + P( B Ç C ) - 2 P( B Ç C ) = p (7.5)
Solution: Let and P(C ) + P( A) - P(C Ç A) = p (7.6)
W1 = Event of drawing a white ball in the first draw. Adding Eqs. (7.4)–(7.6), we have
B1 = Event of drawing black ball in the first draw.
W2 = Drawing white ball in the second draw. 2[ P( A) + P( B) + P(C ) - P( A Ç B)
Now
- P( B Ç C ) - P(C Ç A)] = 3 p (7.7)
W2 = W2 Ç (W1 È B1 ) = (W2 Ç W1 ) È (W2 Ç B1 )
Worked-Out Problems 519
P( A È B) = P( A Ç B) å
r =0
n
Cr 2n-r = nC0 2n + nC1 2n-1 + nC2 2n- 2 + + nCn 20
= 1 - P( A Ç B) = (2 + 1)n = 3n
= 1 - P( A)P( B / A) (Theorem 7.3) Therefore the required probability is
Therefore Statement II is also true. Hence both I n
3n æ 3 ö
and II are true. =ç ÷
4n è 4 ø
Answer: (A)
Answer: (D)
34. Purse A contains 9 coins of 50 paise denomination and
a one rupee coin. Another purse B contains 10 coins 36. n + 1 consecutive natural numbers, if three
of 50 paise denomination. 9 coins are selected at numbers are selected at random, the probability that
random from A and transferred to B. Again 9 coins they are in Arithmetic progression is
are selected at random from B and transferred to A. 3n 3n
(A) (B)
The probability that the rupee coin is still in purse A is 4 n2 - 1 4 n2 + 1
11 10 9 7 2n 2n
(A) (B) (C) (D) (C) (D)
19 19 19 19 4 n2 - 1 4 n2 + 1
520 Chapter 7 Probability
Solution: The procedure ends in first two tests if either and hence
both are faulty or both are good. Therefore the prob-
5 13
ability is P( A Ç B) = 1 - =
18 18
2 1 2 1 1
´ + ´ = Now
4 3 4 3 3
Answer: (A) P( A) = P( A) + P( A Ç B Ç A)
1 1 2n+1 + (-1)n
pn + pn-1 = pn-1 + pn- 2 =
2 2 3× 2n
1 Answer: (D)
= pn- 2 + pn- 3
2
44. In Problem 43, the probability that his score is
1 exactly 5 is
= pn- 3 + pn- 4
2 21 21 65 31
(A) (B) (C) (D)
…………… 32 64 96 96
…………… Solution: In the formula,
1 2n+1 + (-1)n
= p2 + p1 pn =
2 3× 2n
Since put n = 5 so that
1
p1 = P(T ) = p5 =
63
=
21
2 3× 32 32
and p2 = P((T Ç T ) È H ) Answer: (A)
= P(T )P(T ) + P( H )
45. A pair of fair dice is rolled. The probability that the
1 1 1 3 sum is 9 given that 5 appeared on the first die is
= ´ + =
2 2 2 4 1 8 1 5
(A) (B) (C) (D)
we have 9 9 6 6
1 1 3 1 Solution: Let
pn + pn-1 = p2 + p1 = + = 1 E1 = Event of 5 occurs on the first die.
2 2 4 4
E2 = Sum of the faces is 9.
Therefore
Favourable cases to E1 are
2 1 1
pn - = - pn - 1 (5, 1), (5, 2), (5, 3), (5, 4), (5, 5) and (5, 6)
3 3 2
1æ 2ö Favourable cases to E2 are
= - ç pn - 1 - ÷
2è 3ø (5, 4), (4, 5), (3, 6) and (6, 3)
Worked-Out Problems 523
Now
Try it out Six-digit numbers are formed
f randomly
using the digits 0, 1, 2, 3, 4 and 6 with repetitions. If a 1
number is selected at random from these, what is the = P(E )
2
probability that the sum of the digits is 30 with 0 not in
the units place? = P( A)P( B)P(C ) + P( A)P( B)P(C ) + P( A)P( B)P(C )
(This question is another form of Question 48.) æ 1ö æ 1ö æ 1ö
= pq ç ÷ + p(1 - q) ç ÷ + pq ç ÷
è 2ø è 2ø è 2ø
49. The number of throws with a single fair die required
Therefore
for a person such that his chance of getting the face
6 is 1/2 is pq + p(1 - q) + pq = 1
(A) 6 (B) 5 (C) 4 (D) 3 p + pq = 1
Solution: Let x be the number of chances required. Answer: (B)
1
E = Event of throwing 6 so that P(E) =
6 51. Four friends put their car keys on a table. When
Therefore they leave the place, they picked up their keys at
random. The probability that no person picks his
1 5 own key is
P(E ) = 1 - =
6 6 3 1 5 3
(A) (B) (C) (D)
x 8 4 8 4
æ 5ö
Probability of not throwing 6 in all the x chances = ç ÷
è 6ø Solution: This problem is nothing but derangement
Therefore x
problem (See Theorem
T 6.20 and Try
T it out, page 295,Vol.
V 1).
æ 5ö 1 The number of derangements of 4 distinct elements is
Probability of throwing 6 at least once = 1 - ç ÷ =
è 6ø 2
This implies æ 1 1 1 1ö
4 ç 1 - + - + ÷ = 12 - 4 + 1 = 9
è 1 2 3 4ø
log(1/ 2)
x= = 3.8 approximately
log(5 / 6)
When a pair of dice is rolled, the favourable cases to the 54. The probability that India winning a hockey match
event of the sum is 7 or 8 are against Pakistan is 1/2. In a 5-match series, India
surely wins the third match is
(1, 6), (6, 1), (2, 5), (5, 2), (3, 4),
1 1 2 1
(4, 3), (4, 4) (5, 3), (3, 5), (2, 6), (6, 2) (A) (B) (C) (D)
8 4 3 2
Therefore Solution: India winning the third match is independent
11 2 of the results of the first two matches. Hence, the prob-
P(E / H ) = and P(E /T ) = ability that India surely winning the third match is 1/2.
36 11
Answer: (D)
Therefore from Eq. (7.8),
55. The probability of India winning a test match against
1 11 1 2
P(E ) = ´ + ´ England is 1/2. In a five match series, the probability
2 36 2 11 that India registers its second win in the third test is
121 + 72 193 (you can assume the independence from match to
= =
72 ´ 11 792 match)
Answer: (B) 1 1 1 2
(A) (B) (C) (D)
8 2 4 3
53. The odds in favour of a book reviewed by three
independent critics are, respectively, 5 : 2, 4 : 3 and Solution: India has to win one of the first two tests and
3 : 4. The probability that majority of the critics give a win in the third test. Therefore required probability is
favourable remark is P(W Ç L Ç W ) + P(L Ç W Ç W ) = P(W )P(L)P(W )
210 209 211 205 + P(L)P(W )P(W )
(A) (B) (C) (D)
343 343 343 343
1 1 1 1 1 1
= ´ ´ + ´ ´
Solution: Let E1, E2 and E3 be the events of the critics 2 2 2 2 2 2
giving favourable remarks. Then 1
=
5 4 3 4
P(E1 ) = , P(E2 ) = and P(E3 ) =
7 7 7 where W denotes win and L denotes loss.
E be the event that majority reviewed favourably. Answer: (C)
Therefore
56. A person has to go through three successive tests.
E = (E1 Ç E2 Ç E3 ) È (E1 Ç E2 Ç E3 ) The probability of his passing first test is p. If he fails
È (E1 Ç E2 Ç E3 ) È (E1 Ç E2 Ç E3 ) in one of the tests, then the probability his passing
next test is p/2, otherwise it remains the same. For
Hence selection, the person must pass atleast two tests. The
probability that the person to be selected is
P(E) = P(E1 )P(E2 )P(E3 ) + P(E1 )P(E2 )P(E3 )
(A) p2 + p3 (B) 22p2 - p3
+ P(E1 )P(E2 )P(E3 ) + P(E1 )P(E2 )P(E3 ) 1 3
(C) p2 - 2p
2 3 (D) p2 + p
5 4 æ 3ö æ 5ö 4 3 5 2
= ´ ´ ç1 - ÷ + ç1 - ÷ ´ ´ +
7 7 è 7ø è 7ø 7 7 7 Solution: Let Ei (i = 1, 2, 3) be the event of the person
æ 4ö 3 5 4 3 passing the ith test and E is the event that he is selected.
´ ç1 - ÷ ´ + ´ ´ Then
è 7ø 7 7 7 7
5 4 4 2 4 3 5 3 3 5 4 3 E = (E1 Ç E2 ) È (E1 Ç E2 Ç E3 ) È (E1 Ç E2 Ç E3 )
= ´ ´ + ´ ´ + ´ ´ + ´ +
7 7 7 7 7 7 7 7 7 7 7 7 Therefore
80 + 24 + 455 + 60 P(E) = P(E1 )P(E2 / E1 ) + P(E1 )P(E2 / E1 )P(E3 / E2 )
=
7´7´7
+ P(E1 )P(E2 / E1 )P(E3 / E2 )
209
= p p
343 = p × p + p(1 - p) + (1 - p) × p
2 2
Answer: (B)
= 2 p2 - p3
Answer: (B)
526 Chapter 7 Probability
57. A and B are independent events and C is the event It is given that
in which exactly one of A or B occurs. Then
1
(A) P(C ) £ P( A È B)P( A Ç B) P(defective screw of A) =
10
(B) P(C ) ³ P( A È B)P( A Ç B)
1
(C) P(C ) = P( A È B)P( A Ç B) and P(defective screw of B) =
5
(D) P(C) = P(A È B) P(A Ç B)
Therefore
Solution: We have
P(E) = P(E1 ) + P(E2 ) + P(E3 ) + P(E4 ) + P(E5 ) (7.10)
C = ( A - B) È ( B - A) = ( A Ç B) È ( A Ç B)
Now
Since A and B are independent, the pairs ( A, B), ( A, B)
and ( A, B) are also pairs of independent events. Further 4
C1 ´ 5C1 1 4 2
A Ç B and A Ç B are mutually exclusive. Now, let P(E1 ) = 9
´ ´ =
C2 10 5 45
P(A) = x and P(B) = y. Therefore
5
C1 ´ 4 C1 1 9 1
P(C ) = P( A Ç B) + P( A Ç B) P(E2 ) = 9
´ ´ =
C2 5 10 10
= P( A)P( B) + P( A)P( B) 5
C1 ´ 4 C1 1 1 1
P(E3 ) = 9
´ ´ =
= x(1 - y) + (1 - x) y (7.9) C2 10 5 90
4 2
Also C2 æ 1 ö 1
P(E4 ) = ´ç ÷ =
9
C2 è 10 ø 600
P(A È B)P(A Ç B) = [ P(A) + P(B) - P(A Ç B)]P(A)P(B)
5 2
C2 æ 1 ö 1
= ( x + y - xy)(1 - x)(1 - y) P(E5 ) = ´ç ÷ =
9
C2 è 5 ø 90
= ( x + y)(1 - x)(1 - y) - xy(1 - x)(1 - y)
Substituting these values in Eq. (7.10), we have
£ ( x + y)(1 - x)(1 - y) [∵ xy(1 - x)(1 - y) ³ 0]
303
= x(1 - x)(1 - y) + y(1 - x)(1 - y) P(E ) = = 0.168 (approximately)
1800
= x(1 - y) - x2 (1 - y) + y(1 - x) - y2 (1 - x) Answer: (A)
= x(1 - y) + y(1 - x) - [ x2 (1 - y) + y2 (1 - x)] 59. If a circle is selected at random touching all the sides
of a triangle, then the probability that it touches two
£ x(1 - y) + y(1 - x) = P(C ) [by Eq
q. (7.9)]
sides externally and one side internally is
Answer: (B)
1 3 1 2
(A) (B) (C) (D)
4 4 2 3
58. A factory A produces 10% of defective screws and
another factory B produces 20% of defective screws. Solution: It is known that there are four circles touching
A bag contains 4 screws of factory A and 5 screws of all the three sides of a triangle out of which one is the
factory B. If two screws are drawn at random from incircle and the other three are ex-circles. Therefore the
the bag, then the probability that atleast one screw is required probability is 3/4.
defective (correct to three places of decimals) is Answer: (B)
(A) 0.168 (B) 0.158 (C) 0.165 (D) 0.155
Solution: Let E be the event that atleast one screw is 60. Three groups A, B and C are competing for the posi-
defective. Then E can be regarded as the union of the tions on the Board of Directors of a company. The
following events. probabilities of their winning are 0.5, 0.3 and 0.2,
respectively. If the group A wins, the probability of
E1 = one defective from A and one non-defective from B.
introducing a new product is 0.7 and other corre-
E2 = one defective from B and one non-defective from A. sponding probabilities for groups B and C are, respec-
E3 = one defective from A and one defective from B. tively, 0.6 and 0.5. The probability that new product
E4 = both defective from A. will be introduced is
E5 = both defective from B. (A) 0.43 (B) 0.53 (C) 0.63 (D) 0.73
Worked-Out Problems 527
Solution: Let A, B and C be the events winning of A,B and 62. Bag A contains 5 red and 7 white balls and bag B
C, respectively. E is the introduction of new product. Then contains 3 red and 12 white balls. One of the bags
is selected at random and one ball is drawn from it.
E = E Ç ( A È B È C ) = (E Ç A) È (E Ç B) È (E Ç C ) The probability that the drawn ball is red is
Therefore 37 83 63 17
(A) (B) (C) (D)
P(E) = P(E Ç A) + P(E Ç B) + P(E Ç C ) 120 120 120 120
= P( A)P(E / A) + P( B)P(E / B) + P(C )P(E /C ) Solution: Let A and B denote the events of selecting
bags A and B, respectively, and R denote drawing a red
5 7 3 6 2 5 ball. Then
= ´ + ´ + ´
10 10 10 10 10 10
R = ( A È B) Ç R = ( A Ç R) È ( B Ç R)
63
= = 0.63 Therefore
100
Answer: (C) P(R) = P( A Ç R) + P( B Ç R)
= P( A)P(R / A) + P( B)P(R / B)
61. A box contains m green balls and n yellow balls. It is
given that the probability of drawing 2 yellow balls 1 5 1 3
= ´ + ´
from the bag is 5 times the probability of drawing 2 2 12 2 15
green balls. Also, the probability of drawing 1 ball
1æ 5 3ö
of each colour is equal to 6 times the probability of = ç + ÷
drawing 2 green balls. Then the pair (m, n) is equal to 2 è 12 15 ø
(A) (6, 3) (B) (3, 5) (C) (4, 6) (D) (3, 6) 25 + 12 37
= =
2 ´ 60 120
Solution: Let GK and YK be the events of drawing K
green balls and K yellow balls. Therefore Answer: (A)
and P(G1 Ç Y1 ) = 6 P(G2 ) Þ 2 mn = 6 m(m - 1) (7.12) Solution: Let E1 and E2 be the events of the boy
watching DOORDARSHAN and TEN SPORTS, respec-
From Eq. (7.12), we have n = 3(m -1). Put this value of n tively. It is given that
in Eq. (7.11) so that
1 4
3(m - 1)(3m - 4) = 5m(m - 1) P(E1 ) = and P(E2 ) =
5 5
4 m2 - 16 m + 12 = 0 E be the event of the boy falls asleep. Again by
(m - 1)(m - 3) = 0 hypothesis
m = 1, 3 3 1
P(E / E1 ) = and P(E / E2 ) =
4 4
Now
Now
m n = 3m - 3
1 0 E = E Ç (E1 È E2 ) = (E1 Ç E) È (E2 Ç E)
3 6 so that
Since n ¹ 0, the pair (m, n) = (3, 6). P(E) = P(E1 )P(E / E1 ) + P(E2 )P(E / E2 )
Answer: (D)
528 Chapter 7 Probability
64. The chance that doctor A will diagnose disease X P( H2 ) = P(E1 Ç H1 )P( H2 /(E1 Ç H1 )) + P(E1 Ç T1 )
correctly is 60%. The chance that a patient of doctor P( H2 /(E1 Ç H1 )) + P(E2 Ç H1 )P( H2 /(E2 Ç H1 ))
A dies after correct treatment is 75% while it is 80%
after wrong diagnosis. A patient of doctor A having = P(E1 )P( H1 /E1 )P( H2 /(E1 Ç H1 )) + P(E1 )P(T1 /E1)
disease X dies. The probability that his disease is P( H2 /(E1 Ç T1 )) + P(E2 )P( H1 /E2 )
correctly diagnosed is
P( H2 /(E2 Ç H1 )) (Corollary 7.1)
8 9 11 6
(A) (B) (C) (D)
17 17 17 17 1 1 1 1 1 1 1
= ´ ´1+ ´ ´ + ´1´
2 2 2 2 2 2 2
Solution: Let A denote the event of correct diagnosis
and E the event of patient’s death. It is given that 1 1 1 5
= + + =
4 8 4 8
60 40
P( A) = , P( A) = , Answer: (A)
100 100
75 80 66. In a test, an examinee either guesses or copies or
P(E / A) = and P(E / A) =
100 100 knows the answer for a multiple choice question
having FOUR choices of which exactly one is correct.
By Bayes’ theorem (Theorem 7.7)
The probability that he makes a guess is 1/3 and the
P( A)P(E / A) probability for copying is 1/6. The probability that
P( A / E ) = his answer is correct, given that he copied it is 1/8.
P( A)P(E / A) + P( A)P(E / A)
The probability that he knew the answer, given that
60 75 his answer is correct is
´
= 100 100 5 9 24 20
60 75 40 80 (A) (B) (C) (D)
´ + ´ 29 29 29 29
100 100 100 100
3 3 Solution: Let the events be defined as
´ E1: Guessing
= 5 4
3 3 2 4 E2: Copying
´ + ´
5 4 5 5 E3: Knowing
9 E: Correct answer
=
17 By hypothesis,
Answer: (B)
1 1 1 1 1
P(E1 ) = , P(E2 ) = , P(E3 ) = 1 - - =
65. A person has two coins in his pocket of which one is 3 6 3 6 2
a fair coin and the other has heads on both the sides 1
P(E / E1 ) = (out of four choicces only one is correct)
(i.e., two headed coin). He selects one of the coins at 4
random and tosses. If head appears, then he will toss
1
the other coin otherwise he tosses the same coin. The P(E / E2 ) =
probability that head appears in the second toss is 8
5 3 7 1 P(E / E3 ) = 1
(A) (B) (C) (D)
8 8 8 8
Solution: Let P(E3)P(E /E3)
P(E3 /E) =
H1: First toss head. P(E1)P(E /E1) + P(E2)P(E /E2) + P(E3)P(E /E3)
T1: First toss tail.
Worked-Out Problems 529
70. A bag contains 4 black, 2 white and 6 red balls. Solution: Let Bj be the number of black balls trans-
Another bag contains 3 black and 5 white balls. An ferred (j = 0, 1, 2, 3). B is the event of drawing a black
unbiased die is thrown. If either 1 or 2 appears, a ball ball. Therefore
is chosen from the first bag, otherwise a ball from 5
the second bag. If the drawn ball is black then the C4 5
P( B0 ) = 8
=
probability that 2 appeared on the die is C4 70
2 11 6 7
(A) (B) (C) (D) 3
C1 ´ 5C3 30
13 13 13 13 P( B1 ) = 8
=
C4 70
Solution: Let the events be
3
C2 ´ 5C2 30
E1: Event of 1 appearing. P( B2 ) = 8
=
C4 70
E2: Event of 2 appearing.
E3: E1 È E2 = E1 Ç E2.
3
C3 ´ 5C1 5
P( B3 ) = 8
=
B: Event of drawing a black ball. C4 70
Now Also
1 1 4 P( B / B0 ) = 0 (∵ no black ball is transferred)
P(E1 ) = , P(E2 ) = , P(E3 ) =
6 6 6
1
4 P( B / B1 ) =
P( B / E1 ) = 4
12
2
4 P( B / B2 ) =
P( B / E2 ) = 4
12
3
3 P( B / B3 ) =
P( B / E3 ) = 4
8
Therefore by Bayes’ theorem,
By Bayes’ theorem
P( B3 )P( B / B3 )
P(E2)P(B /E2) P( B3 / B) =
P(E2 /B) = 3
P(E1)P(B /E1) + P(E2)P(B /E2) + P(E3)P(B /E3) å P(B )P(B / B )
i =0
i i
1 4
´ 5 3
= 6 12 ´
1 4 1 4 4 3 = 70 4
´ + ´ + ´ 5 30 1 30 2 5 3
6 12 6 12 6 8 ´0+ ´ + ´ + ´
70 70 4 70 4 70 4
1
3 1 6 2 15 15 1
= = ´ = = = =
1 1 3 3 13 13 30 + 60 + 15 105 7
+ +
3 3 2 Answer: (D)
Answer: (A)
72. A person goes to office either by car, scooter, bus
71. Bag A contains 5 white and 3 black balls. Bag B is or train whose probabilities are, respectively, 1/7, 3/7,
empty. Four balls are taken at random from A and 2/7 and 1/7. The probability that he reaches office on
transferred to empty bag B. From B, a ball is drawn time, if he takes car, scooter bus or train is 7/9, 8/9, 5/9
at random and is found to be black. Then, the prob- and 8/9, respectively. Given that he reached office in
ability that among the transferred balls three are time, the probability that he travelled by car is
black and one is white is 1 2 6 5
(A) (B) (C) (D)
1 7 6 1 7 7 7 7
(A) (B) (C) (D)
8 8 7 7
Solution: Let E1, E2, E3 and E4 denote the events
of the person travelling by car, scooter, bus or train,
Worked-Out Problems 531
respectively. Let E denote the event of his reaching 74. If a natural number n is selected from the set {4, 5,
office in time. By hypothesis 6, …, 23}, then the probability that
1 3 2 1 x1 x2 x3 x4 + x2 x3 x4 x5 + x3 x4 x5 x6 + + xn x1 x2 x3 = 0
P(E1 ) = , P(E2 ) = , P(E3 ) = , P(E4 ) =
7 7 7 7 where each of xj is either 1 or -1 is
7 8 1 2 1 3
P(E / E1 ) = , P(E / E2 ) = (A) (B) (C) (D)
9 9 5 5 4 4
5 8 Solution: Out of 20 numbers, one is selected in 20 ways.
P(E / E3 ) = , P(E / E4 ) =
9 9 Let E be the event that
By Bayes’ theorem x1 x2 x3 x4 + x2 x3 x4 x5 + + xn x1 x2 x3 = 0
P(E1 )P(E / E1 ) where each xj = 1 or -1 (j = 1, 2, …, n). Let
P(E1 / E) = 4
å P(E )P(E / E )
j =1
j j y1 = x1 x2 x3 x4 , y2 = x2 x3 x4 x5 , yn = xn x1 x2 x3
so that
1 7
´ y1 + y2 + y3 + + yn = 0
= 7 9
1 7 3 8 2 5 1 8
´ + ´ + ´ + ´ and each of yj = 1 or -1 (∵ xj = ±1). Therefore n must
7 9 7 9 7 9 7 9 be even say n = 2m. Among yj, m are +1 and m are -1.
7 1 Therefore
= =
49 7 y1 y2 y3 yn = (-1)m (1)m = (-1)m (7.13)
Answer: (A)
But
73. a is selected from the first 100 y1 y2 y3 yn = x14 x24 x34 xn4 = 1 (∵ xj = ± 1)
natural numbers. The probability that
From Eq. (7.13), (-1)m = 1 and m = 2K. So
éaù éaù éaù
a=ê ú+ê ú+ê ú n = 2m = 4K
ë2û ë3û ë5û
Therefore n is a multiple of 4. Among the numbers 4, 5,
where for any real x, [x] denotes the integral part of 6, …, 23 there are 5 multiples of 4. Hence
x, is
(A) 0.4 (B) 0.3 (C) 0.2 (D) 0.1 5 1
P(4) = =
20 4
Proof: The natural number “a” can be selected from
the 100 numbers in 100 ways. Let Answer: (C)
30 60 3
= 0.3 P( H ) = =
100 100 5
Answer: (B) 2
P( H ) =
5
532 Chapter 7 Probability
P(W4 /W ) =
P(W4 )P(W /W4 ) Probability = 1
4
å P(W )P(W /W )
j =1
j j
Answer: (D)
Answer: (D)
r × a £ 12 Û x + y + z £ 12 (7.14)
Worked-Out Problems 533
The number of possible integral solutions satisfying the 81. An urn contains 5 red balls and 3 blue balls. A ball is
inequality Eq. (7.14) is (Theorem 6.15, Vol. 1) selected at random and discarded, but 2 more balls
12
of the other colour are added into the bag. A second
å ( n - 1)
C2 = 2C2 + 3C2 + 4C2 + + 11C2 ball is selected at random. Then the probability that
n= 3 this ball is red is
= (3C3 + 3C2 ) + (4C2 + 5C2 + + 11C2 ) 31 13 41 23
(A) (B) (C) (D)
(∵ C3 = 1 = C2 )
3 2 72 36 72 36
P(E1 )P( B / E1 ) 86. Let X be a random variable whose range is {-1, 0, 1}.
P(E1 / B) =
P(E1 )P( B / E1 ) + P(E2 )P( B / E2 ) If mean m of X is 0.1, P(X = 0) = 0.3, then P(X = 1) is
3 1 (A) 0.2 (B) 0.4 (C) 0.32 (D) 0.3
´
= 5 2 Solution: By Definition 7.17
3 1 2 3
´ + ´ m = å xk P( X = xk )
5 2 5 10
Worked-Out Problems 535
å P( X = K ) = 1
4
K =1 å P ( X = x) = 1
x =1
Þ 3l 3 - 10 l 2 + 9 l - 1 = 1 Þ 10 K = 1
Þ 3l 3 - 10 l 2 + 9 l - 2 = 0 1
ÞK=
Þ (l - 1)(3l - 1)(l - 2) = 0 10
P(2 £ X £ 3) = P( X = 2) + P( X = 3) m = å xP( X = x)
æ 1ö æ 1ö 5 = 1(K ) + 2(2 K ) + 3(3K ) + 4(4 K )
= 4 ç ÷ - 10 ç ÷ + - 1
è 3ø è 9ø 3
30
= 30 K = =3
10 8 10
=- +2=
9 9
s 2 is the variance, then by Quick Look 2
Answer: (A)
s 2 + m 2 = å x2 P ( X = x)
88. The range of a random variable is the set of all natural
numbers and l is a positive constant. If = 12 (K ) + 22 (2 K ) + 32 (3K ) + 42 (4 K )
100
lk = 100 K = = 10
P( X = k ) = (k = 1, 2, 3, …) 10
k
Therefore
then l equals
s 2 = 10 - 32 = 1
1 1
(A) loge 2 (B) loge 3 (C) (D) loge 3 Answer: (B)
2 2
Solution: We have 90. Let X be a binomial variate with parameters n and p.
¥
If the mean is 20 and variance is 15, then p is equal to
l l2 l3
å
K =1
P ( X = K ) = 1 Þ
1
+
2
+
3
++ ¥ = 1
(A)
1
(B)
1
(C)
1
(D)
3
3 4 2 4
536 Chapter 7 Probability
Solution: Mean m = np and variance s 2 = npq 93. X is a Poisson variate such that
(see Theorem 7.10). Therefore
2
P( X = 2) = P( X = 1)
np = m = 20 3
and 15 = npq then P(X = 3) is
Solving we get 34 -3/ 4 36 -4 / 3
(A) e (B) e
81 81
15 = 20 q
32 -4 / 3 36 -3/ 4
3 (C) e (D) e
Þq= 81 81
4
Solution: Let m be the parameter of the Poisson distri-
3 1
Þ p=1- q =1- = bution. Then
4 4
e- m mk
Answer: (B) P( X = k ) =
k
91. X is a binomial variate with mean 10 and variance 5.
Then the number of trials is equal to Hence
(A) 10 (B) 15 (C) 20 (D) 16 2
P ( X = 2) = P( X = 1)
3
Solution: Let n be the number of trial and p the prob-
ability of a success. Then by Theorem 7.10 e- m × m2 2 - m
Þ = e ×m
2 3
10 = np, 5 = npq
4
Hence Þm=
3
1
5 = 10q Þ q = and so
2
1 e-4 / 3 (4 / 3)3 64 -4 / 3 32 -4 / 3
and p=1- q = P( X = 3) = = e = e
2 3 27 ´ 6 81
So Answer: (C)
one of the coins at random and tossed it three times 97. The probability of a coin showing head is p. 100 coins
and observed 2 heads and 1 tail. The probability that are tossed at a time. If the probability of 50 coins
the selected coin is A is showing heads is same as 51 coins showing heads,
7 18 9 16 then the value of p is
(A) (B) (C) (D)
25 25 25 25 49 51 1 49
(A) (B) (C) (D)
100 101 2 101
Solution: Let A, B and C denote the events of selecting
the coins A, B and C, respectively, so that Solution: By hypothesis,
P( A) = P( B) = P(C ) =
1 100
C50 p50 q50 = 100C51 p51 q49
3
where q = 1 - p. Therefore
Let E be the event of getting 2 heads and 1 tail. Hence by
the binomial distribution we have 100 100
(q) = ( p)
50 50 51 49
2
æ 1ö æ 1ö 3
P(E / A) = 3C2 ç ÷ ç ÷ = q p
è 2ø è 2ø 8 =
50 51
2
æ 2 ö æ 1ö 4 51q = 50 p
P(E / B) = C2 ç ÷ ç ÷ =
3
è 3 ø è 3ø 9
51(1 - p) = 50 p
2
æ 1ö æ 2 ö 2 51
P ( E /C ) = 3 C2 ç ÷ ç ÷ = p=
è 3ø è 3ø 9 101
Therefore by Bayes’ theorem Answer: (B)
96. A man takes a step forward with probability 0.4 and P(at least five successes) = P( X = 5) + P( X = 6)
a step backward with probability 0.6. After 11 steps, 5 6 0
æ 2 ö æ 1ö æ 2 ö æ 1ö
the probability that he is one step away from the = 6C5 ç ÷ ç ÷ + 6C6 ç ÷ ç ÷
è 3 ø è 3ø è 3 ø è 3ø
starting point is
(A) 462 ´ (0.24)5 (B) 462 ´ (0.25)5 6 ´ 32 64 256
= + =
(C) 362 ´ (0.24)5 (D) 362 ´ (0.25)5 729 729 729
Answer: (B)
Solution: One step away means, six forward and five
backward steps or six backward and five forward steps. 99. A book writer writes a good book with probability
Therefore 1/2. If it is a good book, the probability that it will
P(one step away) = C6 (0.4) (0.6) + C6 (0.6) (0.4)
11 6 5 11 6 5 be published is 2/3, otherwise it is 1/4. If he writes
2 books, the probability that at least one book will
= 11C6 (0.4)5 (0.6)5 (0.4 + 0.6) be published is
= 462 ´ (0.24)5 407 411 405 307
(A) (B) (C) (D)
Answer: (A) 576 576 576 576
538 Chapter 7 Probability
Solution: Let 15 6 1
= + +
G = Event of good book 64 64 64
G ¢ = Event of not a good book 22 11
= =
E = Event of publication 64 32
Then Answer: (D)
+ P( X = 6) Solution: We have
9
p = Probability for a good bulb =
4 2 5 6 0
æ 1ö æ 1ö æ 1ö æ 1ö æ 1ö æ 1ö
= 6 C 4 ç ÷ ç ÷ + 6 C 5 ç ÷ ç ÷ + 6 C6 ç ÷ ç ÷ 10
è 2ø è 2ø è 2ø è 2ø è 2ø è 2ø
1
q = Probability for a defective bulb =
10
Worked-Out Problems 539
=
56 ´ 95 å P( X = k ) = 1
k =0
108
Answer: (A)
¥
e-2 2k
Þå =1
k =0 k
103. A random variable X has Poisson distribution such
that P(X = 1) = P(X = 2). If m and s 2 are the mean Therefore
and variance of the distribution, then
æ 2 22 ö
m å P( X ³ 3) = 1 - e -2
çè 1 + +
1 2 ÷ø
=
s2
5
1 =1-
(A) 2 (B) (C) 1 (D) 2 e2
2 Answer: (B)
Solution: Let l be the parameter of X
X. By hypothesis
105. If the mean of a Poisson distribution is 1/2 then the
P( X = 1) = P( X = 2) ratio of P(X = 3) is to P(X = 2) is
-l
e l e l
1 -l 2 (A) 1: 6 (B) 1: 8 (C) 1: 4 (D) 1: 2
Þ =
1 2 Solution: For a Poisson distribution, the mean is equal
to the parameter. Hence the parameter is 1/2. Now,
Þl=2
e-1/ 2 (1/ 2)3
For a Poisson distribution, mean and variance are equal P( X = 3) =
(see Theorem 7.11) 3
= P ( M DN ) P ( E ) - P ( E )P (F )
= [ From Eq. (7.17)]
P(F )
= P((M - N ) È ( N - M ))
= P(M - N ) + P(N
N - M) P(E)(1 - P(F ))
=
P(F )
= P(M ) - P(M Ç N ) + P( N ) - P(M Ç N )
P ( E )P (F )
[part (2), Theorem 7.2] = = P(E )
P(F )
= P(M ) + P( N ) - 2P(M Ç N )
Hence (B) is correct.
Hence (A) is correct. Also, (C) follows from (B) and so (C) is correct. Now,
P(M Ç N ) + P(M Ç N ) = P(M - N ) + P( N - M ) P (E Ç F ) P (E Ç F )
P (E / F ) + P (E / F ) = +
= P(M DN ) P(F ) P(F )
Solution: We have 1 - P( A È B)
=
3 1 1 - P( A)
P( A) = =
9 3 1 - P( A È B)
=
1 1 - (0.4)
P(E / A) =
2 1 - P( A È B)
=
Therefore 0.6
1 1 1 Therefore
P(E Ç A) = ´ =
3 2 6 P( A È B) = 1 - (0.6)(0.6)
So (A) is correct. Now = 1 - 0.36 = 0.64
4 So (A) is correct. Now
P(C ) = and P(E /C ) = 1
9
P( A Ç B) = P( A)P( B / A)
Therefore
= (0.4)(0.9) = 0.36
4 4
P(C Ç E) = ´ 1 = Hence (B) is correct. Again
9 9
So (B) is correct. Now the event E is given by 0.64 = P( A È B)
= P( A) + P( B) - P( A Ç B)
E = ( A È C ) Ç E = ( A Ç E) È (C Ç E)
= 0.4 + P( B) - (0.36)
Therefore
Therefore
P(E) = P( A Ç E) + P(C Ç E)
P( B) = 1 - (0.4) = 0.6
1 4 11
= + =
6 9 18 Given P(B) we have
P( A Ç B)
P( A / B) =
P( B)
P(C Ç E)
P(C/ E) =
P(E ) 0.36
= = 0.6
0.6
4 11 8
= ¸ =
9 18 11 So (C) is correct. Again
So (D) is correct. P( A Ç B)
P( A / B) =
Answers: (A), (B), (D) P( B)
P( A) - P( A Ç B)
5. A, B are two events with positive probabilities. If =
P(A) = 0.4, P( B / A) = 0.9, P( B / A) = 0.6, then 1 - P( B)
1 1
(A) P( A) = (C) P(C Ç A) =
4 10
1 37
(B) P( B) = (D) P( A È B È C ) =
18 50
1 Solution: We have
(C) P( A Ç B) =
18 Number of even numbers £ 100 is equal to 50.
2 Number of multiples of 3 £ 100 is 33.
(D) P(exactly one of A and B) =
9
Number of multiples of 5 £ 100 is 20.
Solution: For the event A, the favourable cases are Number of common multiples of 2 and 3 is 16.
(2, 2), (2, 4), (2, 6), Number of common multiples of 3 and 5 is 6.
(4, 2), (4, 4), (4, 6), Number of common multiples of 2 and 5 is 10.
(6, 2), (6, 4), (6, 6) Number of common multiples of 2, 3 and 5 is 3.
Therefore Now,
9 1 50 33 20
P( A) = = P( A) = , P( B) = , P(C ) =
36 4 100 100 100
Hence (A) is correct. For the event B, the favourable 16 6 10
cases are (4, 6), (5, 5) and (6, 4). Therefore P( A Ç B) = , P( B Ç C ) = , P(C Ç A) =
100 100 100
3 1 Also
P( B) = =
36 12
P( A È B È C )
So (B) is not correct. Now
= P( A) + P( B) + P(C ) - P( A Ç B) - P( B Ç C )
A Ç B = {(4, 6), (6, 4)}
- P(C Ç A) + P( A Ç B Ç C )
This implies
50 33 20 16 6 10 3
= + + - - - +
2 1 1000 100 100 100 100 100 100
P( A Ç B) = =
36 18 106 - 32 74 37
= = =
So (C) is correct. Finally 100 100 50
P(exactly one of A and B is correct) Hence all (A), (B), (C) and (D) are correct.
Answers: (A), (B), (C), (D)
= P(( A - B) È ( B - A))
Solution: Let A and B denote the events “A belongs Solution: Since there are 4 even numbers among
to the committee” and “B belongs to the committee,” 1 to 9, it follows that
respectively. Then
4
9
P(E / H ) =
C 36 3 9
P( A) = 10 2 = =
C3 120 10
Again, since there are 2 even numbers among 1 to 5, it
3 follows that
P( B) =
10 2
P(E /T ) =
8
C1 8 1 5
P( A Ç B) = 10
= =
C3 120 15 Therefore (A) and (B) are correct. Now,
Now
E = ( H È T ) Ç E = ( H Ç E) È (T Ç E)
P( A È B) = P( A) + P( B) - P( A Ç B)
This implies
3 3 1 16 8
= + - = = P(E) = P( H Ç E ) + P(T Ç E )
10 10 15 30 10
= P( H )(E / H ) + P(T )P(E /T ) (Theorem 7.3)
So
2 4 1 2 58
P(exactly one of A and B) = P(( A - B) È ( B - A)) = ´ + ´ =
3 9 3 5 135
= P( A - B) + P( B - A)
So (C) is correct. By Bayes’ theorem,
8 8
C C
= 10 2 + 10 2 P ( H )P (E / H )
C3 C3 P( H / E ) =
P( H )P(E / H ) + P(T )P(E /T )
28 28
= + 2 4
´
120 120 3 9
=
56 2 4 1 2
= ´ + ´
120 3 9 3 5
8 135 20
=
7 = ´ =
15 277 58 29
Hence all (A), (B), (C) and (D) are correct. Hence (D) is correct.
Answers: (A), (B), (C), (D) Answers: (A), (B), (C), (D)
2 ´ 3 ´ 3 72 1 = 1 - P( A)P( B)
p2 = = =
6 720 10 Consider the following example which shows P(AÈB) ¹
Hence (C) is correct. Finally 1 - P( A)P( B) when A Ç B = f.
Let S be the sample space obtained when a fair die
2´ 3´ 3 1 is rolled so that S = {1, 2, 3, 4, 5, 6}. Let A = {1, 3, 5} and
p2¢ = =
6 10 B = {2, 4, 6} so that A Ç B = f and P(A È B) = P(S) = 1.
But
(Example: b1b2b3g1g2g3 or g1g2g3b1b2b3.) Therefore (D) is
1
not correct. P( A) = P( B) = = P( A) = P( B)
2
Answers: (A), (B), (C)
Therefore
11. Let A and B be any two events in a sample space.
Then 1 1
1 - P( A)P( B) = 1 - ´
(A) When P(B) ¹ 0, 2 2
3
P( A) + P( B) - 1 =
P( A / B) ³ 4
P( B)
¹ P( A È B)
is always true
(B) P( A Ç B) = P( A) - P( A Ç B) is always true Hence (D) is not correct.
Answers: (A), (B), (C)
(C) P( A È B) = 1 - P( A)P( B) if A and B are
independent 12. A and B are two random events such that 0 < P(A) < 1
(D) P( A È B) = 1 - P( A)P( B) if A and B are and 0 < P(B) < 1. Then which of the following are
disjoint. true?
Solution: We have (A) P(A/B) > P(A) Þ P(B/A/ ) > P(B)
P( A Ç B) (B) P( B / A) + P( B / A) = 1
P( A / B) =
P( B) (C) P( A / B) = P( A / B) only when A Ç B = f
P( A) + P( B) - P( A È B) (D) P( B / A) = P( B / A) only when A and B are
=
P( B) independent
Worked-Out Problems 545
Solution: Suppose (C) is also false for which consider the same sample space:
S = {1, 2, 3, 4, 5, 6} and let A = {2, 4, 6} and B = {2, 3, 5}.
P( A / B) > P( A) Then
P( A Ç B)
Þ > P( A) 2
P( B) P( A / B) =
3
P( A Ç B)
Þ > P( B) [∵ 0 < P( A), P( B)] P( A / B) =
2
P( A) 3
Þ P( B / A) > P( B)
but A Ç B ¹ f. Finally
So (A) is correct. We now show that (B) is false. P( B / A) = P( B / A)
Consider the following example: Let S = {1, 2, 3, 4, 5, 6}
which is obtained by rolling a fair die. Let A = {2, 6} and P( A Ç B) P( B Ç A)
Û =
B = {2, 3, 5}. Then P( A) P( A)
1 Û [1 - P( A)]P( A Ç B) = P( A)P( B Ç A)
P( A / B) =
3 Û [1 - P( A)]P( A Ç B) = P( A)[ P( B) - P( A Ç B)]
1
P( A / B) = Û P( A Ç B) = P( A)P( B)
3
Û A and B are independent events
so that
Hence (D) is correct.
2
P( A / B) + P( A / B) = ¹ 1 Answers: (A), (D)
3
2. In a class, there are 10 boys and 5 girls. Three students 3. A and B are two events with P(A) = 1/3, P(B) = 1/4
are selected one after other without allowing the and P(A È B) = 1/2. Match the items of Column I
earlier selected student to join the class. Match the with those of Column II.
items of Column I with the items of Column II.
Column I Column II
Column I Column II
1
9 (A) P(A/B) is equal to (p)
(A) The probability that the first two (p) 3
are boys and the third a girl is 13 1
3 (B) P(B/A
/ ) equals (q)
(B) The probability that first and third
T (q) 4
are boys and the second a girl is 7 1
5 (C) The value of P( A Ç B) is (r)
(C) The probability that first and third (r) 2
are of same sex and the second is of 21 2
(D) P( A / B) is equal to (s)
opposite sex is 3
15
(D) If the first and third are of same sex (s)
and the second is of opposite sex, 91 Solution: We have
then probability that the second
1
student is girl is = P( A È B)
2
Solution: Let b and g denote boy and girl, respectively. = P( A) + P( B) - P( A Ç B)
(A) The probability that the first two are boys and the 1 1
third a girl is = + - P( A Ç B)
3 4
10 9 5 15 Therefore
P(bbg ) = ´ ´ =
15 14 13 91
1 1 1
Answer: (A) Æ (s) P( A Ç B) = + -
3 4 2
The probability that first and third are boys and the
second a girl is 4+3-6 1
= =
12 12
10 5 9 15
P(bgb) = ´ ´ =
15 14 13 91 (A) We have
Answer: (B) Æ (s) P( A Ç B) 1 1 1
P( A / B) = = ¸ =
(C) The probability that first and third are of same sex P( B) 12 4 3
and the second is of opposite sex is
Answer: (A) Æ (p)
P ((bgb) È ( gbg )) = P(bgb) + P( gbg ) (B) We have
15 5 10 4 P( B Ç A) 1 1 1
= + ´ ´ P( B / A) = = ¸ =
91 15 14 13 P( A) 12 3 4
15 20 Answer: (B) Æ (q)
= +
91 3 ´ 91
(C) We have
65 5
= =
3 ´ 91 21 P( A Ç B) = P( A - B)
Answer: (C) Æ (r) = P( A) - P( A Ç B)
Let E1 = (bgb) È (gbg) and E2 = bgb. Then
1 1
= -
P(E2 ) 3 12
P(E2 / E1 ) =
P(E1 )
3 1
= =
15 21 9 12 4
= ´ =
91 5 13 Answer: (C) Æ (q)
Answer: (D) Æ (p)
Worked-Out Problems 547
(D) We have 2 5
´
= 3 16
P( A Ç B)
P( A / B) = 2 5 1 3
´ + ´
P( B) 3 16 3 8
1/ 4 1 10 5
= = = =
1 - (1/ 4) 3 10 + 6 8
Answer: (D) Æ (p)
Answer: (D) Æ (p)
4. Bag X contains 5 red, 3 white and 8 black balls. Bag Y
contains 3 red and 5 white balls. A fair die is tossed. If 5. , B and C are three students of Mathematics. Each
2 or 5 appears a ball from bag Y is chosen, otherwise a is given two problems in probability. Their chances
ball from bag X is chosen. Match the items of Column I of solving the problems are, respectively, 1/2, 1/3
with those of Column II. Here R, W and B denote and 1/6. They try independently. Let A1, A2, B1, B2,
drawing red ball, white ball and black ball, respectively. C1, C2 denote the events of their solving the first and
second problems. Match the items of Column I with
those of Column II.
Column I Column II
5 Column I Column II
/X) =
(A) P(R/X (p)
8
1
3 (A) P(A1 Ç B2) (p)
Y =
(B) P(R/Y) (q) 4
16
1
3 (B) P(C1 Ç A2) (q)
W/X) =
(C) P(W/ (r) 6
8
1
5 (r)
X R) =
(D) P(X/ (s) 12
16 (C) P(B1 Ç C2)
1
(s)
Solution: We have 18
1
4 2 (D) P(A1 Ç A2) (t)
P( X ) = = 36
6 3
2 1 Solution:
P(Y ) = =
6 3 (A) The required probability is
(A) The required probability is P( A1 Ç B2 ) = P( A1 )P( B2 )
5 1 1 1
P(R / X ) = = ´ =
16 2 3 6
Answer: (A) Æ (s) Answer: (A) Æ (q)
(B) The required probability is (B) The required probability is
3 P(C1 Ç A2 ) = P(C1 )P( A2 )
P ( R /Y ) =
18
1 1 1
Answer: (B) Æ (r) = ´ =
6 2 12
(C) The required probability is
Answer: (B) Æ (r)
3 (C) The required probability is
P(W / X ) =
16
Answer: (C) Æ (q) P( B1 Ç C2 ) = P( B1 )P(C2 )
(D) By Bayes’ theorem, 1 1 1
= ´ =
3 6 18
P ( X )P (R / X )
P( X / R) = Answer: (C) Æ (s)
P( X )P(R / X ) + P(Y )P(R /Y )
548 Chapter 7 Probability
P( X £ 2) = P( X = 0) + P( X = 1) + P( X = 2) 3 1 15
npq = 5 ´ ´ =
4 4 16
Answer: (D) Æ (q)
Comprehension-Type Questions
1. Passage: We consider a two-player game in which 2 7 2 1
there is always a winner. A, B and C are three players. (A) (B) (C) (D)
9 9 3 3
2
If A plays B, then the probability of A beating B is . (ii) When B plays C, the winner plays A. Then, the
3 probability that A is the final winner is
2 2 5 7 4
If B plays C, then the probability of B beating C is . (A) (B) (C) (D)
3 9 9 9 9
2
If C plays A, then the probability of C beating A is . (iii) When C plays A, the winner plays B. Then, the
3
probability that A is the final winner is
Answer the following questions:
5 4 2 7
(i) When A plays B, the winner plays C. Then, the (A) (B) (C) (D)
9 9 9 9
probability that A is the final winner is
Worked-Out Problems 549
Solution: Therefore
(i) First, A must beat B and then A must beat C. The 4 1
probability that A is the final winner is = 1 + P( B) Þ P( B) =
3 3
2 1 2 Answer: (B)
´ =
3 3 9 (ii) The required probability is
Answer: (A)
P( A Ç B)
(ii) Whether B beats C or C beats B, the winner must P( A / B) =
P( B)
lose to A. Therefore
Probability (A is final winner) P( A)P( B)
=
= P(B beats C and loses to A) P( B)
+ P(C
C beats B and loses to A) 1
= P( A) =
2 2 1 1 5 2
= ´ + ´ =
3 3 3 3 9 Answer: (D)
Answer: (B) (iii) A and B are independent. This implies A and B are
(iii) First, A must beat C and then A must beat B. The also independent. So
probability that A is the final winner is
P( B Ç A)
P( B / A) =
1 2 2 P( A)
´ =
3 3 9
P( B)P( A)
Answer: (C) =
P( A)
1 2 = 1 - P( B)
P( A) = and P( A È B) =
2 3 1 2
= 1- =
Answer the following questions. 3 3
(i) P(B) is equal to Answer: (B)
1 1 2 1
(A) (B) (C) (D) 3. Passage: A box contains 3 coins out of which two are
2 3 3 4
fair coins and the third is a two headed coin. A coin
(ii) P(A/B) equals is selected at random and tossed. If head appears, the
3 2 1 1 same coin is tossed again. If tail appears, another is
(A) (B) (C) (D) selected from the remaining two coins and tossed.
4 3 3 2
Answer the following questions.
(iii) P( B / A) is equal to (i) The probability that head appears twice is
1 2 1 2 1 2 1 1
(A) (B) (C) (D) (A) (B) (C) (D)
2 3 3 5 2 3 3 4
(i) The required probability is (iii) If the drawn ball is white, then the probability
2
that it is from urn C is
æ 5 ö æ 1 ö 25
P( X = 3) = q2 p = ç ÷ ç ÷ = 2 3 12 13
è 6 ø è 6 ø 216 (A) (B) (C) (D)
5 5 25 25
Answer: (A)
(ii) The required probability is Solution:
(i) Let W denote that drawn ball is white. Then
P( X ³ 3) = q2 p + q3 p + q4 p + + ¥
W = (A È B È C) Ç W
= q2 p(1 + q + q2 + + ¥)
= ( A Ç W ) È ( B Ç C ) È (C Ç A)
æ 5ö æ 1ö æ ö 25
2
1
=ç ÷ ç ÷ç = Therefore
è 6 ø è 6 ø è 1 - (5 / 6) ÷ø 36
P(W ) = P( A Ç W ) + P( B Ç W ) + P(C Ç W )
Answer: (B)
= P( A)P(W / A) + P( B)P(W / B) + P(C )P(W /C )
(iii) The required probability is (∵ X > 3)
2 2 2 1 1 4
P(( X ³ 6)/( X > 3)) = 1 - ( P( X = 4) + P( X = 5)) = ´ + ´ + ´
5 5 5 5 5 5
= 1 - ( p + pq) 10 2
= =
= 1 - p(1 + q) 25 5
Answer: (C)
1æ 5ö
= 1 - ç1 + ÷ (ii) Let E denote that drawn ball is black. Then
6è 6ø
P(E) = P( A)P(E / A) + P( B)P(E / B) + P(C )P(E /C )
11 25
=1- = 2 3 2 4 1 1
36 36 = ´ + ´ + ´
Answer: (D) 5 5 5 5 5 5
15 3
= =
6. Passage: Urns A, B, C, respectively, contain 2 white 25 5
and 3 black balls, 1 white and 4 black balls, 4 white and Answer: (D)
1 black balls. The probabilities of choosing the urns
(iii) By Bayes’ theorem we have
are, respectively, 2/5, 2/5, and 1/5. One of the urns is
chosen at random and a ball is drawn from it. Answer P(C )P(W /C )
the following questions: P(C /W ) =
P( A)P(W / A) + P( B)P(W / B) + P(C )P(W /C )
(i) Probability that the drawn ball is white is
1 4
12 13 2 3 ´
(A) (B) (C) (D) = 5 5
25 25 5 5 2 2 2 1 1 4
´ + ´ + ´
(ii) Probability that the drawn ball is black is 5 5 5 5 5 5
13 12 2 3 4 2
(A) (B) (C) (D) = =
25 25 5 5 10 5
Answer: (A)
3
æ 1ö æ 1ö æ 1ö
=4´ç ÷ ç ÷ çè∵ p = q = ÷ø
è 2ø è 2ø 2
The animal escapes when the sportsman misses in all the By hypothesis
five shots. Therefore the probability of animal escaping
23
to jungle is P( B) =
90
6 æ 1ö æ 1 öæ 1öæ 1 öæ 1öæ 1ö
Õ ç1 - 2 ÷ = ç1 - 2 ÷ ç1 - 3 ÷ ç1 - 4 ÷ ç1 - 5 ÷ ç1 - 6 ÷ Therefore
j=2 è j ø è 2 øè 2 øè 2 øè 2 øè 2 ø
3n + 4(n + 4) 23
éæ 1ö æ 1ö æ 1ö æ 1ö æ 1ö ù =
= êç 1 - ÷ ç1 - ÷ ç1 - ÷ ç1 - ÷ ç1 - ÷ 18(n + 4) 90
ëè 2ø è 3ø è 4ø è 5ø è 6 ø úû
35n + 80 = 23(n + 4)
éæ 1ö æ 1ö æ 1ö æ 1ö æ 1ö ù
´ êç 1 + ÷ ç1 + ÷ ç1 + ÷ ç1 + ÷ ç1 + ÷ 12 n = 12
ëè 2ø è 3ø è 4ø è 5ø è 6 ø úû
n=1
æ 1 2 3 4 5ö æ 3 4 5 6 7 ö
=ç ´ ´ ´ ´ ÷ç ´ ´ ´ ´ ÷ Answer: 1
è 2 3 4 5 6ø è 2 3 4 5 6ø
1 7 7 p 6. A and B are two independent events whose probabili-
= ´ = =
6 2 12 q ties are, respectively, 1/n and 1/(n + 1). If the proba-
bility of A Ç B is 1/12, then n equals .
Therefore
Solution: A and B are independent events. This implies
q - p = 12 - 7 = 5
P( A Ç B) = P( A)P( B)
Answer: 5
Therefore
5. Boxes B1, B2, B3 contain different coloured balls as
1
given in Table 7.9. The probabilities of selecting boxes = P( A)P( B)
are, respectively, 1/6, 1/2 and 1/3. One of the boxes is 12
chosen at random and a ball is drawn from it. If the 1
=
probability of the drawn ball is black is 23/90 then the n(n + 1)
value of n is equal to .
which gives
Table 7.9 Integer answer type question 5 (n is
a positive integer) n=3
Answer: 3
White Black Red
B1 2 n 2 7. A number x is selected from the set of first 9 natural
numbers (i.e., x = 1, 2, 3, …, 9). If the probability that
B2 3 2 4
f x)) = x where
ff( f(
B3 4 3 2
f ( x) = x2 - 3 x + 3
Solution: Let B be denote the event of drawing a black is m/9, then m is equal to .
ball. Then
Solution: Clearly all the solutions of f( f x) = x are also
B = ( B1 È B2 È B3 ) Ç B solutions of f( f x)) = x. First, we solve f(
f f( f x) = x.
= ( B1 Ç B) È ( B2 Ç B) È ( B3 Ç B) f ( x) = x Þ x2 - 3 x + 3 = x
Therefore Þ x2 - 4 x + 3 = 0
Þ ( x - 1)( x - 3) = 0
P( B) = P( B1 )P( B / B1 ) + P( B2 )P( B / B2 ) + P( B3 )P( B / B3 )
Þ x = 1, 3
1 n 1 2 1 3
= ´ + ´ + ´
6 n+4 2 9 3 9 Therefore x = 1, 3 are also solutions of f( f x)) = x. We
f f(
n want to seek if there are any more solutions of f( f x)) = x
f f(
2
= + other than 1 and 3.
6(n + 4) 9
f ( f ( x)) = x Þ f ( x2 - 3 x + 3) = x
3n + 4(n + 4)
=
18(n + 4) Þ ( x2 - 3 x + 3)2 - 3( x2 - 3 x + 3) + 3 = 0
Worked-Out Problems 555
Þ x4 - 6 x3 + 12 x2 - 9 x + 3 = 0 since
Þ ( x2 - 4 x + 3)( x2 - 2 x + 1) = 0 1
P( P1 pairing with X ) =
4
Þ ( x - 1)( x - 3)( x - 1)2 = 0
as there are four pairs.
Þ x = 1, 3
P(P1 to lose in II) = P(P1 wins in I)P(P1 pairing with X in II)
the probability that x satisfies equation f( f x)) = x is 2/9.
f f( P( P1 losing)
Therefore m = 2.
1 1 1
Answer: 2 = ´ ´
2 2 2
8. A fair coin is tossed n times. Let X denote the number 1
=
of heads appeared. If P(X = 4), P(X = 5) and P(X = 6) 8
are in AP, then the smallest values of n is .
P( P1 to lose in III) = P( P1 wining I and II) P( P1 loosing)
Solution: Since the coin is fair,
æ 1 1ö 1
1 =ç ´ ÷ ´
P( H ) = P(T ) = è 2 2ø 2
2
1
By binomial distribution, =
8
n- K K
æ 1ö æ 1ö
P ( X = K ) = n CK ç ÷ çè ÷ø Therefore probability of P1 losing to X is
è 2ø 2
1 1 1 3
æ 1ö
n + + =
= CK ç ÷
n 8 8 8 8
è 2ø
Hence
By hypothesis
n-m=8-3=5
2 P( X = 5) = P( X = 4) + P( X = 6) Answer: 5
Therefore
10. The odds against an event A is 2 :3 and odds in
2(nC5 ) = nC4 + nC6 favour of another event B is 1:2. If A and B are
independent and P( A È B) = m / n, then | m - n| is
n2 - 21n + 98 = 0
. Here m and n do not have proper common
n = 7, 14 divisor.
SUMMARY
7.1 Random experiment: An experiment is called 7.8 Usual probability: The classical probability is also
random experiment if the following conditions are called usual probability.
satisfied.
(1) The experiment can be repeated any number of 7.9 Sample points and sample space: Any possible
times under similar conditions. outcome of a random experiment is called a sample
point and the set of all sample points is called
(2) All possible outcomes of the experiment are
the sample space of the random experiment. An
known in advance
elementary event means a sample point. Generally
(3) The actual outcome in a particular experiment sample space is denoted by S.
cannot be exactly predicted.
7.10 Finite sample space: A set A is called finite if either
7.2 Sample space and event: The set of all possible A is an empty set or it is bijective with the set {1,
outcomes of a random experiment is called sample 2, 3, …, n} for some positive integer n. If a sample
space of the experiment and any subset of the sample space is finite, then it is called a finite sample space.
space is called an event.
7.11 Countably infinite sample space: A set A is called
7.3 Exhaustive events: Two or more events are called countably infinite set if it is bijective with the set
exhaustive events if the performance of the experi- + of all positive integers. If the sample space of a
ment results in the occurrence of at least one of random experiment is countably infinite set, then
these events. the sample space is called countably infinite. For
example, tossing a fair coin till head appears has a
7.4 Mutually exclusive events: Two or more events are countably infinite sample space.
said to be mutually exclusive if the occurrence of one
of the events prevents the occurrence of any one of 7.12 Definition: Here afterwards events mean subsets
the other events. of the sample space. If A and B are two events, then
A È B means at least one of A or B and A Ç B
7.5 Equally likely events: Two or more events are said means both A and B. Impossible event is denoted
to be equally likely (or equiprobable) if there is no by empty set f and a certain event means the entire
reason to expect one of them in preference to the sample space.
others.
7.13 Various events in set theoretical form:
7.6 Probability (classical definition): Suppose in a
(1) Events E1, E2, …, En are said to be mutually
random experiment there are n exhaustive, mutually
exclusive if Ei Ç Ej = f for i ¹ j.
exclusive, equally likely outcomes. If m of them are
favourable to an event E, then the probability P(E) (2) E1, E2, …, En are called exhaustive events if E1 È
of E is defined as E2 È È En = S (sample space).
(3) Mutually exclusive and exhaustive events means
m
P(E ) = E1 È E2 È È En = S and Ei Ç Ej = f for i ¹ j.
n
(4) E1, E2, …, En are called equally likely (or equi-
probable) if there is no reason to expect one of
QUICK LOOK
them in preference to the others.
0 £ P(E) £ 1 (∵ 0 £ m £ n)
7.14 Probability function (Axiomatic): Let S be the
sample space connected with a random experi-
ment and Ã(S) is the power set of S (i.e., the set
7.7 Complementary event: If E is an event, then non-
of all subsets of S). Then a function P : Ã(S) ®
occurrence of E is called complementary eventt of E
is called a probability function on the sample S, if it
and is denoted by E (or Ec).
satisfies the following conditions:
(1) P(E) ³ 0 for all E ÎÃ(S)
QUICK LOOK
(2) P(S) = 1
n-m m (3) If E1 and E2 belong to Ã(S) and E1 Ç E2 = f, then
P(E ) =
c
= 1 - = 1 - P(E )
n n
P(E1 È E2 ) = P(E1 ) + P(E2 )
Summary 557
If P is a probability function on S, then for any 7.18 Conditional probability: Let A and B be two events
E ÎÃ(S), P(E) is called the probability of the event E. and P be a probability function with P(B) > 0, then
If E = {s}, then we write P(s) instead of P({s}). we define P( A Ç B)/ P( B) as the conditional prob-
If S is a countably infinite space then (3) will be replaced ability of the occurrence of event A after the occur-
by the following: rence of the event B and it is denoted by P( A / B).
(3¢) If { En } is a sequence of mutually exclusive events, That is
then
P( A Ç B)
¥ ¥ P( A / B) =
P æ È En ö = å P(En )
P( B)
è n=1 ø n=1
QUICK LOOK
provided the infinite sum exists.
If the sample space S is finite, then
QUICK LOOK Number of favourable cases to both A and B
P( A /B) =
One can verify that the usual probability (classical Number of favourablee cases to B
definition) is also a probability function according to
modern probability definition given above in 7.14.
7.19 Multiplication theorem: Let A and B be two events
of a random experiment with positive probabilities,
7.15 Theorem: The following hold good for any prob- then
ability function P defined on a sample space S. P( A)P( B / A) = P( A Ç B)
(1) P(f) = 0
= P( B)P( A / B)
(2) P(E ) = 1 - P(E )
(3) 0 £ P(E) £ 1 for all E ÎÃ(S) 7.20 Corollary: Let E1, E2, …, En be n events with posi-
n-1
(4) P(E1 - E2) = P(E1) - P(E1 Ç E2) and in particular tive probabilities and P æ Ç Ei ö > 0, then
P(E1 - E2) = P(E1) - P(E
E2) whenever E2 Ì E1 and è i =1 ø
hence E2 Ì E1 Þ P(E1) ³ P(E E2).
P(E1 Ç E2 Ç Ç En ) = P(E1 )P(E2 / E1 )P(E3 /(E1 Ç E2 ))
(5) If E1, E2, …, En are mutually exclusive events,
n-1
then P(En /( Ç Ei ))
i =1
P(E1 È E2 È È En ) = P(E1 ) + P(E2 ) + + P(En )
7.21 Definition (independent events): Two events A
7.16 Definition (Odds in favour, odds against): If E is and B are said to be independent of each other if
an event, E is its complementary event and P is a the occurrence of one does not effect the occur-
probability function, then the ratio P(E): P(E) is rence of the other. That is, B is independent of A,
called odds in favour of E and the ratio P(E): P(E) if P( B / A) = P( B).
is called odds against E.
7.22 Theorem: Two events A and B are independent if
7.17 Addition theorem: If A and B are two events of a and only if P(A Ç B) = P(A)P(B).
random experiment, then
7.23 Theorem (total probability): Let E1, E2, …, En be
P( A È B) = P( A) + P( B) - P( A Ç B)
mutually exclusive and exhaustive events with posi-
and in particular, if A Ç B = f, then tive probabilities. Then for any event E,
P( A È B) = P( A) + P( B)
n
P(E) = å P(Ej )P(E / Ej )
j =1
QUICK LOOK
7.24 Bayes’ theorem: Let E1, E2, …, En be mutually
If A, B and C are any three events, then
exclusive and exhaustive events with positive prob-
P( A È B È C ) = P( A) + P( B) + P(C ) abilities. If E is an event then
- P( A Ç B) - P( B Ç C ) P(EK )P(E / EK )
P(EK / E) = n
- P(C Ç A) + P( A Ç B Ç C ) å P(E )P(E / E )
j j
j =1
558 Chapter 7 Probability
7.29 Definition (discrete random variable): A random 7.34 Binomial variate: Let n be the number of indepen-
variable X is called discrete if its range is finite or dent Bernoulli trials in which the probability for
countably infinite. a success is p and that of failure is q = 1 - p. If X
denotes the number of successes, then
7.30 Definition (mean and variance): Let X be a P( X = k ) = n Ck pk qn - k
random variable on a sample space S and suppose
the range of X = {x1, x2, …, xn, …}. Let P be a prob- P(X = k) is called the probability of k successes and
ability function on S. Let P(X = xn) = P(X X-1{xn}). n – k failures. n and p are called parameters of the
Then binomial variate X and we write X ~ B(n, p).
(1) If å x P( X = x ) is finite, then it is called the
n
n n
7.35 Theorem (mean and variance of X ~ B (n, p)):
mean of X and is denoted by mX or simply m If X ∼ B(n, p), then the mean of X is np and the
when there is no confusion about X
X. That is variance is npq.
m = å xn P( X = xn )
n QUICK LOOK
7.36 Poisson distribution: Let X be a random variable 7.37 Theorem: The mean and variance of a Poisson
whose range is {0, 1, 2, 3, …, n, …} and l a positive variate X with parameter l are equal and equal
number. We define to l.
Note: In Vol. 3 (Calculus) we will prove that the Poisson
e- l l k
P( X = k ) = distribution is a limiting case of binomial distribution.
k
EXERCISES
Single Correct Choice Type Questions
1. A bag contains 8 white, 6 black, 2 red and 4 green 8. An ellipse of eccentricity 2 2 / 3 is inscribed in a
balls. Two balls are picked at random at a time. The circle. A point inside the circle is selected at random.
probability that they are of different colours is The probability that the point lies outside the
7 9 14 12 ellipse is
(A) (B) (C) (D)
19 19 19 19 2 1 1 3
(A) (B) (C) (D)
3 3 2 4
2. An urn contains 6 white, 4 red and 10 green marbles.
Three are drawn one after other without replacement. 9. Two subsets A and B are chosen at random from
The probability that they are of different colours is a set containing 6 elements one after other with
2 3 6 2 replacement. The probability that A and B contain
(A) (B) (C) (D) equal number of elements is
9 19 19 57
131 231 331 101
(A) (B) (C) (D)
3. A bag contains 4 black and 7 white balls. Two balls 1021 1021 1021 1021
are drawn randomly. The probability that at least one
of them is black is 10. A lot contains 12 cell phones out of which 4 are
(A) 0.71 (B) 0.618 (C) 0.728 (D) 0.628 defective. If two cell phones are drawn from the lot
at random, the probability that one is defective and
4. Three fair coins are tossed at a time. The probability the other is non-defective is
of getting at least one head is 16 15 13 17
(A) (B) (C) (D)
7 1 3 4 33 33 33 33
(A) (B) (C) (D)
8 8 8 8
11. On the real line (i.e., x-axis) points p and q are
5. There are 9 books of which one is very popular. The selected at random such that -2 £ q £ 3 and 0 £ p £ 3.
books are arranged in a row at random. The probability The distance between p and q is greater than 3 with
that the popular book is always at the middle place is probability
2 1 (4!)2 2(4!)2 2 1 1 3
(A) (B) (C) (D) (A) (B) (C) (D)
9 9 9! 9! 3 3 4 4
6. The letters of the word PECULIAR are arranged 12. A and B are two events such that
at random. The probability that all the vowels are 3 1 1
together is P( A) = , P( B) = and P( A Ç B) =
8 2 4
3 1 1 3
(A) (B) (C) (D) Then P( A Ç B) is equal to
7 7 14 14
1 1 1 1
(A) (B) (C) (D)
7. A and B are two students among six students. If all 12 3 16 4
the six students are arranged in a row, the probability
that B immediately succeeds A is 13. A bag contains 4 red, 3 white, 2 blue and 1 green
1 1 2 2 marbles. Four marbles are drawn from it at random.
(A) (B) (C) (D)
5 6 5 3
560 Chapter 7 Probability
The probability that the draw contains one each 1/2, 3/4 and 1/4. They try the problem independently.
from the given colours is The probability that majarity of them solving the
1 6 4 31 problem is
(A) (B) (C) (D)
7 7 35 35 1 1 2 3
(A) (B) (C) (D)
3 2 3 4
14. A gardner plants 3 apple trees, 4 neem trees and
5 mango trees in a row at random. The probability 22. A lot contains 15 cameras out of which 4 are defec-
that no two mango trees are next to one another is tive. Cameras are drawn one after other to identity
7 1 8 1 the defective pieces. The process continues till all
(A) (B) (C) (D) the defective pieces are identified. The probability
99 9 99 11
that the proceedure ends at the ninth testing is
15. The first 11 letters of the English alphabet are arranged 11 13 8 9
(A) (B) (C) (D)
in all possible ways at random. The probability that 195 195 195 195
there are exactly 4 letters in between A and B is
12 14 7 6 23. The probability that man aged x years to die in five
(A) B) (C) (D) years is 1/3. Out of persons P1, P2, P3, P4 and P5, each
55 55 55 55
aged x years, P1 will die in 5 years and he is the first
person to die in 5 years is
16. Two positive real numbers x and y such that x + y =
12 are selected at random. The probability that xy 211 311 111 1
(A) (B) (C) (D)
is greater than or equal to 3/4 times their greatest 1215 1315 1115 5
product is
1 1 2 3 24. The probabilities of three mutually exclusive events are
(A) (B) (C) (D)
3 2 3 4 1 + 3p 1 - p 1 - 2 p
, ,
3 4 2
17. From 15 consecutive positive integers, 3 are selected
at random. The probability that they are in AP is Then
3 7 9 11 1 1 1
(A) (B) (C) (D) (A) 0 < p £ (B) £ p£
65 65 65 65 3 3 2
1 1 1
18. Five numbers are selected at random from the first 50 (C) < p<1 (D) £ p £
2 5 4
natural numbers and arranged in the increasing order
x1 < x2 < x3 < x4 < x5. The probability that x3 is 30 is 25. Two events A and B are such that P(A) = 0.7,
29
C2 ´ 20C2 39
C2 ´ 19C2 P(B) = 0.4 and P( A Ç B) = 0.5. Then P( B /( A È B))
(A) (B)
50
C5 50
C5 is equal to
30
C2 ´ 20C2 55 1 1 1 2
(C) (D) (A) (B) (C) (D)
50
C5 1029 2 3 4 3
19. Two different coloured dice are rolled. A is the event 26. A fair die is rolled four times. Out of the four face
that the sum of the faces appeared is odd and B is values obtained, the probability that the minimum
the event that one of the faces turned up shows face value is greater than or equal to 2 and the
face 1. Then P( A Ç B) is maximum value is less than or equal to 5 is
1 2 1 3 16 1 8 65
(A) (B) (C) (D) (A) (B) (C) (D)
3 3 4 4 81 81 81 81
20. Two dice are thrown at a time. The probability that 27. A purse contains four 50 paise coins and three
the sum is greater than 8 is Rs. 1 coins. Another purse contains six 50 paise coins
and two Rs. 1 coins. One of the purses is selected
1 1 1 5
(A) (B) (C) (D) random and a coin is drawn from it. The probability
6 9 3 18 that it is a 50 paise coin is
35 25 37 27
21. A problem in combinatorics is given to three students (A) (B) (C) (D)
whose probabilities of solving it are, respectively, 56 56 56 56
Exercises 561
28. If each of a, b and c takes values from the set {1, 2, is in a police court. The probability that they have
3, 4, 5, 6}, then the probability that the equation given identical statements is
ax2 + bx + c = 0 has real roots is equal to 5 1 7 2
(A) (B) (C) (D)
53 43 23 63 12 3 12 3
(A) (B) (C) (D)
216 216 216 216
36. There are five pairs of socks in a cupboard. If 4 socks
29. In a regular hexagon ABCDEF,
F if three vertices are are drawn at random, the probability that the draw
selected at random, then the probability that they contains exactly one matching is
form the vertices of an equilateral triangle is 8 2 5 13
(A) (B) (C) (D)
1 1 2 1 21 7 7 21
(A) (B) (C) (D)
5 3 3 10
37. In a bolt-producing factory, machines A, B and C
30. From the set {1, 2, 3, …, 15}, seven numbers are give 25%, 35% and 40% of the total output, respec-
selected at random one after other with replace- tively. It is also known that 5%, 4% and 2% of them
ment. The probability that the largest of these is 9 is are defective. If a bolt of the factory is selected at
7 7 7 7 random then the probability that it is defective is
æ 8ö æ 9ö æ 7ö æ 3ö
(A) ç ÷ (B) ç ÷ (C) ç ÷ (D) ç ÷ 69 63 73 83
è 15 ø è 10 ø è 10 ø è 5ø (A) (B) (C) (D)
2000 2000 2000 2000
31. Two cards are drawn at random from a deck of
38. A and B are two politicians who settle disputes
52 playing cards one after other, replacing the earlier
drawn card. The probability that the first card is between parties independently. The probabilities of
diamond and the second card is Queen is their settling disputes amicably are, respectively, 4/7
and 7/15. A firm has engaged them for a settlement
3 4 5 1 of dispute between the firm and the employees. The
(A) (B) (C) (D)
13 13 52 52 probability that neither of them settle the dispute is
73 32 8 16
32. Let S be a four-element set. If a two-element subset (A) (B) (C) (D)
105 105 35 35
of S is selected at random, the probability that it
belongs to a partition class of S is
39. S is a five-element set. A subset P of A is selected
2 1 3 4 at random. After inspecting the elements of P,
(A) (B) (C) (D)
9 5 5 5 the elements are restored to S. Again a subset Q
(Hint: See Definition 1.28 and Single Correct Choice of S is selected at random. The probability that P
Type Question 13, Chapter 1, Vol. 1, page 60.) and Q form a partition of S (i.e., P È Q = S and
P Ç Q = f) is
33. Let w ¹ 1 be a cube root of unity. A fair die is 1 1 1 1
(A) (B) (C) (D)
thrown three times. If r1, r2 and r3 are the numbers 2 16 8 32
that appeared on the faces, the probability that
wr1 + wr2 + wr3 = 0 is 40. A set X has 10 elements. Subset A of X is selected at
1 1 2 1 random and then the elements of A are returned to
(A) (B) (C) (D) X. Again a subset B of X is chosen at random. The
X
18 9 9 36
probability that B is disjoint with A is
10 5
34. “A” is one of the six horses entered for a race and it 1 1 æ 3ö æ 3ö
(A) (B) (C) ç ÷ (D) ç ÷
is to ridden by one of the two jockeys B and C. If B 210 25 è 4ø è 4ø
rides A, then all the horses are equally likely to win.
If C rides A, the chances of A’s win will be trebled, 41. Three groups of workers contain 3 women, 1 male;
then, the odds in favour of A is 2 women and 2 male and 3 males and 1 woman. One
(A) 1: 2 (B) 2 :1 (C) 1: 3 (D) 3 :1 worker from each group is selected for a work at
random. The probability that the selection consti-
35. A and B are two persons. A speakes truth 2 out of tutes 2 males and 1 lady is
3 times, while B speaks truth 3 out of 4 times. They 9 3 1 13
(A) (B) (C) (D)
are independent witnesses in an accident case which 32 32 32 32
562 Chapter 7 Probability
42. Two persons A and B each tosses a fair coin thrice. n+2 n+1 m+2 m+1
(A) (B) (C) (D)
The probability that they get same number of heads is 2m+1 2m+1 2n+1 2n+1
5 11 7 9
(A) (B) (C) (D) 47. Three identical bags contain the following composi-
16 16 16 16
tion of balls.
43. If A and B are two independent events in a random Bag I: 2 white, 1 black
experiment then P( A È B) + P( A Ç B) is equal to Bag II: 1 white, 2 black
Bag III: 2 white, 2 black
1 2
(A) 0 (B) (C) 1 (D) The probabilities of selecting the bags are equal.
2 3 One of the bags is selected and a ball is drawn from
it. The drawn ball is found to be black and it is kept
44. Three bags have the following combination of balls. outside. Then the probability of a black ball again is
Bag I: 7 black, 3 white
1 2 2 3
Bag II: 4 black, 6 white (A) (B) (C) (D)
Bag III: 2 black, 8 white 3 3 5 5
The probabilities of selecting bags are, respectively,
1/5, 3/5 and 1/5. One of the bags is chosen at random 48. A fair die is rolled. The probability that first 1 appears
and two balls are drawn from it, one after other at even number of trials is
without replacement. The probability that both balls 6 1 5 5
(A) (B) (C) (D)
are black is 11 6 36 11
4 1 41 8
(A) (B) (C) (D) 49. An experiment has 10 equally likely outcomes. Let
45 9 45 45
A and B be two non-empty events of the experi-
(Hint: Total probability.) ment. If A consists of 4 outcomes, the number of
outcomes B should have such that A and B are inde-
45. Five men out of 100 and 25 women out of 1000 have pendent is
colour blindness. If a colour blind person is selected
at random, the probability that the person is a male is (A) 2, 4 or 3 (B) 3, 6 or 9
1 2 1 1 (C) 4 or 8 (D) 5 or 10
(A) (B) (C) (D)
3 3 2 4
50. If the mean and variance of a Binomial variate X
(Hint: Use Bayes’ theorem.) are, respectively, 2 and 1, then P(X > 1) is
11 5 7 9
46. A fair coin is tossed (m + n) times where m > n. The (A) (B) (C) (D)
16 16 16 16
probability for m consecutive heads is
Comprehension-Type Questions
1. Passage: A and B are two bags. A contains 4 fair coins sides. Two coins are transfered from bag A to bag B
and 3 counterfeit coins while B has 5 fair and 7 coun- and then a coin is taken from the bag B and tossed.
terfeit coins. The counterfeit coins have tail on both Answer the following three questions.
Exercises 565
(i) If both coins transfered from A to B are counter- (i) The proportionality constant is equal to
feit coins, then probability of getting a tail is 1 1 1 1
(A) (B) (C) (D)
3 5 1 23 74 148 112 504
(A) (B) (C) (D)
4 28 2 28
(ii) If E is the event of selecting a biased coin out of
(ii) If tail appears, then the probability of both coins 6 coins, then P(E1 / E) is
transferred from bag A are counterfeit is
1 1 1 2
1 23 23 23 (A) (B) (C) (D)
(A) (B) (C) (D) 266 132 133 133
7 28 153 66
(iii) If the coin selected is found to be biased, then
(iii) If head appears, then the probability of both P(E6 / E) is equal to
coins transferred from bag A are fair is
2 3 5 3
14 21 25 14 (A) (B) (C) (D)
(A) (B) (C) (D) 19 38 38 19
43 43 43 19
4. Passage: There are n urns numbered from 1 to n. The
2. Passage: In a tennis tournament, there are 12 players Kth urn contains K white balls and (n + 1 - K) black
K
S1, S2, …, S12 and they are divided into 6 pairs at random. balls. Let Ek denote the event of selecting the K
Kth urn
From each pair, the winner will be decided on the basis at random and let W denote the event that the ball
of the game played between the two players of the pair. drawn from the selected urn is white. Let P(A) denote
Assume that all players are of equal strength so that the probability of an event A. Answer the following
the probability of any player beating any other player questions.
is 1/2. Answer the following questions.
(i) If P(EK) is proportional to K for K = 1, 2, 3, …, n,
(i) The probability that S1 and S2 are not playing then lim P(W ) is
against other is n®¥
1 10 1 5 1 2 1 3
(A) (B) (C) (D) (A) (B) (C) (D)
2 11 11 22 3 3 4 4
(ii) The probability that exactly one of S1 and S2 is (ii) If P(EK) = C, a constant for all K = 1, 2, 3, …, n,
among the losers is then P(En /W ) is equal to
1 5 6 5 2 1 n 1
(A) (B) (C) (D) (A) (B) (C) (D)
2 22 11 11 n+1 n+1 n+1 2
(iii) The probability that both S1 and S2 are among (iii) If P(EK ) = 1/ n for all K = 1, 2, 3, …, n and E
the winners is denote the event of choosing an even numbered
1 5 6 5 urn, then P(W / E) is equal to
(A) (B) (C) (D)
4 11 11 12 1 n+4
(A) (B)
3. Passage: A box contains 6 coins out of which at least 2n 2(n + 1)
one is biased. Let EK(1 £ K £ 6) denote the event that n+2 n+1
(C) (D)
exactly K out of 6 coins are biased. Also let P(EK) 2(n + 1) 2n
be directly proportional to K(K + 1). Answer the
following questions.
2. Suppose two persons A and B have n + 1 and n coins. Statement I: P(Ei / E) > P(Ei )P(E / Ei )
All the coins are fair. They toss their coins.
n
Statement I: The probability that A gets more heads Statement II: å P(E ) = 1
i =1
i
than B is 1/2.
Statement II: The probability that A gets more heads 5. A random number selector can only select one of the
than B is the same as A gets more tails than B. numbers from the set {1, 2, 3, …, 9} and he will make
these selections with equal probabilities.
3. In throwing a pair of fair dice, let A1 be the event that
the first die turns up odd number, A2 be the event that Statement I: After n trials (n > 1), the probability
the second die turns up odd number and A3 be the that the product of these selected numbers is divisible
event that the sum of the faces turn up is odd. by 5 is
n n n
Statement I: The events A1, A2, A3 are independent. æ 8 ö æ 5ö æ 4 ö
1-ç ÷ -ç ÷ + ç ÷
è 9ø è 9ø è 9ø
Statement II: A1, A2, A3 are pairwise independent.
Statement II: For the product to be divisible by 10,
4. Let E1, E2, …, En be mutually exclusive and exhaus- the selections must contain at least one 5 and one
tive events of a random experiment with P(Ei) > 0 for even number.
i = 1, 2, 3, …, n. Let E be any event connected to the
experiment with 0 < P(E) < 1.
ANSWERS
Single Correct Choice Type Questions
1. (C) 26. (A)
2. (D) 27. (C)
3. (B) 28. (B)
4. (A) 29. (D)
5. (B) 30. (D)
6. (C) 31. (D)
7. (B) 32. (A)
8. (A) 33. (C)
9. (B) 34. (A)
10. (A) 35. (C)
11. (B) 36. (D)
12. (A) 37. (A)
13. (C) 38. (C)
14. (A) 39. (D)
15. (D) 40. (C)
16. (B) 41. (D)
17. (B) 42. (A)
18. (A) 43. (C)
19. (A) 44. (D)
20. (D) 45. (B)
21. (B) 46. (A)
22. (C) 47. (A)
23. (A) 48. (D)
24. (B) 49. (D)
25. (C) 50. (A)